Clin Med Review Block

अब Quizwiz के साथ अपने होमवर्क और परीक्षाओं को एस करें!

ØA 61-year-old woman presents to the emergency room (ER) complaining of cough for 2 weeks. The cough is productive of green sputum and is associated with sweating, shaking chills, and fever up to 102°F (38.8°C). She was exposed to her grandchildren who were told that they had upper respiratory infections 2 weeks ago but now are fine. Her past medical history is significant for diabetes for 10 years, which is under good control using oral hypoglycemics. She denies tobacco, alcohol, or drug use. On examination, she looks ill and in distress, with continuous coughing and chills. Her blood pressure is 100/80 mm Hg, her pulse is 110 beats/min, her temperature is 101°F (38.3°C), her respirations are 24 breaths/min, and her oxygen saturation is 97% on room air. Examination of the head and neck is unremarkable. Her lungs have rales and decreased breath sounds, with dullness to percussion in bilateral bases. Her heart is tachycardic but regular. Her extremities are without signs of cyanosis or edema. The remainder of her examination is normal. A complete blood count (CBC) shows a high white blood cell (WBC) count of 17,000 cells/mm3, with a differential of 85% neutrophils and 20% lymphocytes. Her blood sugar is 120 mg/dL. what is the next diagnostic step?

CXR

ØA 25-year-old African-American man is admitted to your service with the diagnosis of a sickle cell pain episode. He was admitted to the hospital six times last year with the same diagnosis, and he was last discharged 2 months ago. This time, he presented to the emergency department complaining of abdominal and bilateral lower extremity pain, his usual sites of pain. When you examine him, you note he is febrile to 101°F, with respiratory rate 25 bpm, normal blood pressure, HR 100 bpm. Lung examination reveals bronchial breath sounds and egophony in the right lung base. His oxygen saturation on 2 L/min nasal cannula is 92%. Besides the usual abdominal and leg pain, he is now complaining of chest pain, which is worse on inspiration. Although he is tender on palpation of his extremities, the remainder of his examination is normal. His laboratory examinations reveal elevated white blood cell and reticulocyte counts, and a hemoglobin and hematocrit that are slightly lower than baseline. The peripheral smear is shown. what is the next step?

CXR and give abx

•A 45-year-old man presents to the clinic with a cough productive of purulent sputum of 3 weeks' duration. He says that he had just gotten over a cold a few weeks prior to this episode. He occasionally has fevers, and he coughs so much that he has chest pain. He reports having a mild sore throat and nasal congestion. He has no history of asthma or any chronic lung diseases. He denies nausea, vomiting, diarrhea, and any recent travel. He denies any smoking history. On examination, his temperature is 98.6 °F (37.0 °C), pulse is 96 beats/min, blood pressure is 124/82 mm Hg, respiratory rate is 18 breaths/min, and oxygen saturation is 99% on room air. Head, ears, eyes, nose, and throat (HEENT) examination reveals no erythema of the posterior oropharynx, tonsillar exudates, uvular deviations, or significant tonsillar swelling. Neck examination is negative. The chest examination yields occasional wheezes, but normal air movement is noted.what is your next step?

CXR to r/o PNA, bronchodilator, antitussives, analgesics, self-limited, drink water, pt counseling

A 58 yo man comes to your office, because of shortness of breath. He has experienced mild dyspnea on exertion for a few years, but more recently he has noted worsening SOB w/ minimal exercise and the onset of dyspnea at rest. He has difficulty reclining; as a result, he spends the night sitting up in a chair trying to sleep. He reports a cough w/ production of yellowish-brown sputum every morning throughout the year. he denies chest pain, ever, chills, or lower extremity edema. He smoked about two packs of cigarettes per day since age 15. He does not drink alcohol. A few months ago, the patient went to an urgent care clinic for evaluation of his symptoms, and he received a prescription for some inhalers, the names of which he does not remember. He was also told to find a primary care physician for further evaluation. On physical exam, his BP is 135/85, pulse 96bpm, resp 28 bpm, and temp 97.6. He is sitting in a chair, leaning forward w/ his arms braced on his knees. He appears uncomfortalbe w/ labored resp and cyanotic lips. He is using accessory muscles of respiration and chest examinatino reveals wheezes and rhonchi BL, but no crackles. the AP of the chest wall appears increased. Cardiovascular examination reveals distant heart sounds but with a RRR. JVD is NL. Extremities w/o cyanosis, edema, or clubbing. What is the next best diagnostic test?

CXR, echo, ABG, SpO2

ØA 65-year-old man with benign prostatic hypertrophy had been experiencing difficulty with urination, and so he saw his urologist to be evaluated for a transurethral resection of the prostate. As part of the routine preoperative evaluation, he had a complete blood count, but that was found to be abnormal. The procedure was cancelled and he is now referred to the internal medicine clinic for additional evaluation. ØAside from his prostate symptoms, the patient is asymptomatic. He has not experienced any recent fevers, chills, night sweats, arthralgias, or myalgias. His appetite is good and his weight has been stable. He is moderately physically active, plays golf regularly, and has not noted any fatigue or exertional dyspnea. ØOn examination, he is afebrile and normotensive. His conjunctivae are anicteric, and his skin and oral mucosa show no pallor. His chest is clear to auscultation, and his heart is regular without any murmurs. Abdominal exam is normal, no lymphadenopathy. Laboratories show the following results: white blood cell (WBC) count is 56,000 with 90% mature lymphocytes and 10% neutrophils, hemoglobin is 14.8 g/dL, hematocrit is 45%, and platelet count is 189,000/uL. His CMP is normal. what is the most appropriate dx?

Chronic Lymphocytic Leukemia (asx)

ØA 58-year-old woman comes to the office after she experienced a near-fainting spell 1 day ago. She was outside playing tennis when she vomited and felt light-headed. She spent the rest of the day lying down with mild, diffuse, abdominal pain and nausea. She had no fever or diarrhea. She reports several months of worsening fatigue, mild, intermittent, generalized abdominal pain, and loss of appetite with a 10- to 15-lb unintentional weight loss. Her medical history is significant for hypothyroidism for which she takes levothyroxine. She takes no other medications. On examination, her temperature is 99.8°F, heart rate is 102 bpm, blood pressure is 89/62 mm Hg, and normal respiratory rate. She does become light-headed, and her heart rate rises to 125 bpm upon standing with a drop in systolic blood pressure to 70 mm Hg. She is alert and well tanned, with hyperpigmented creases in her hands. Her chest is clear, and her heart rhythm is tachycardic but regular. On abdominal examination, she has normal bowel sounds and mild diffuse tenderness without guarding. Her pulses are rapid and thready. She has no peripheral edema. Initial laboratory studies are significant for Na 121 mEq/L, K 5.8 mEq/L, glucose 52 mg/dL, and creatinine 1.0 mg/dL. what is your next step (labs and intervention)?

Cortisol level! give 5% dextrose

ØA 48-year-old woman calls 911 and is brought to the emergency center complaining of a sudden onset of dyspnea. She reports she was standing in the kitchen making dinner, when she suddenly felt as if she could not get enough air, her heart started racing, and she became light-headed and felt as if she would faint. She denied chest pain or cough. Her medical history is significant only for gallstones, for which she underwent a cholecystectomy 2 weeks previously. The procedure was complicated by a wound infection, requiring her to stay in the hospital for 8 days. She takes no medications regularly, and only takes acetaminophen as needed for pain at her abdominal incision site. ØOn examination, she is tachypneic with a respiratory rate of 28 bpm, oxygen saturation 84% on room air, heart rate 124 bpm, and blood pressure 118/89 mm Hg. She appears uncomfortable, diaphoretic, and frightened. Her oral mucosa is slightly cyanotic, her jugular venous pressure is elevated, and her chest is clear to auscultation. Her heart rhythm is tachycardic but regular with a loud second sound in the pulmonic area, but no gallop or murmur. Her abdominal examination is benign, with a clean incision site without signs of infection. Her right leg is moderately swollen from mid-thigh to her feet, and her thigh and calf are mildly tender to palpation. Laboratory studies including cardiac enzymes are normal; her electrocardiogram (ECG) reveals only sinus tachycardia, and her chest x-ray is interpreted as normal. what is the most appropriate diagnostic step?

D-dimer and angiography (CTA), V/Q scan, CXR (r/o other)

ØA 56-year-old white man comes in for a routine health maintenance visit. He is new to your practice and has no specific complaints today. He has hypertension for which he takes hydrochlorothiazide. He has no other significant medical history. He does not smoke cigarettes, occasionally drinks alcohol, and does not exercise. His father died of a heart attack at age 60 and his mother died of cancer at age 72. He has two younger sisters who are in good health. On examination, his blood pressure is 130/80 mm Hg and his pulse is 75 beats/min. He is 6-ft tall and weighs 200 lb. His complete physical examination is normal. You order a fasting lipid panel, which subsequently returns with the following results: total cholesterol 242 mg/dL; triglycerides 138 mg/dL; high-density lipoprotein (HDL) cholesterol 48 mg/dL; and low-density lipoprotein (LDL) cholesterol 155 mg/dL. ØHis 10-year risk of atherosclerotic cardiovascular disease is 9.3% (based on his age, race, nonsmoking status, controlled hypertension, total cholesterol, and HDL level) what are the indications for statin?

ACC, AHA, NICE w/ calculated risk

ØA 58-year-old woman comes to the office after she experienced a near-fainting spell 1 day ago. She was outside playing tennis when she vomited and felt light-headed. She spent the rest of the day lying down with mild, diffuse, abdominal pain and nausea. She had no fever or diarrhea. She reports several months of worsening fatigue, mild, intermittent, generalized abdominal pain, and loss of appetite with a 10- to 15-lb unintentional weight loss. Her medical history is significant for hypothyroidism for which she takes levothyroxine. She takes no other medications. On examination, her temperature is 99.8°F, heart rate is 102 bpm, blood pressure is 89/62 mm Hg, and normal respiratory rate. She does become light-headed, and her heart rate rises to 125 bpm upon standing with a drop in systolic blood pressure to 70 mm Hg. She is alert and well tanned, with hyperpigmented creases in her hands. Her chest is clear, and her heart rhythm is tachycardic but regular. On abdominal examination, she has normal bowel sounds and mild diffuse tenderness without guarding. Her pulses are rapid and thready. She has no peripheral edema. Initial laboratory studies are significant for Na 121 mEq/L, K 5.8 mEq/L, glucose 52 mg/dL, and creatinine 1.0 mg/dL. what is the test used to confirm the likely diagnosis?

ACTH stim test

ØA 61-year-old woman presents to the emergency room (ER) complaining of cough for 2 weeks. The cough is productive of green sputum and is associated with sweating, shaking chills, and fever up to 102°F (38.8°C). She was exposed to her grandchildren who were told that they had upper respiratory infections 2 weeks ago but now are fine. Her past medical history is significant for diabetes for 10 years, which is under good control using oral hypoglycemics. She denies tobacco, alcohol, or drug use. On examination, she looks ill and in distress, with continuous coughing and chills. Her blood pressure is 100/80 mm Hg, her pulse is 110 beats/min, her temperature is 101°F (38.3°C), her respirations are 24 breaths/min, and her oxygen saturation is 97% on room air. Examination of the head and neck is unremarkable. Her lungs have rales and decreased breath sounds, with dullness to percussion in bilateral bases. Her heart is tachycardic but regular. Her extremities are without signs of cyanosis or edema. The remainder of her examination is normal. A complete blood count (CBC) shows a high white blood cell (WBC) count of 17,000 cells/mm3, with a differential of 85% neutrophils and 20% lymphocytes. Her blood sugar is 120 mg/dL. what is the next step in therapy?

Abx, in/outpt

ØA 58-year-old woman comes to the office after she experienced a near-fainting spell 1 day ago. She was outside playing tennis when she vomited and felt light-headed. She spent the rest of the day lying down with mild, diffuse, abdominal pain and nausea. She had no fever or diarrhea. She reports several months of worsening fatigue, mild, intermittent, generalized abdominal pain, and loss of appetite with a 10- to 15-lb unintentional weight loss. Her medical history is significant for hypothyroidism for which she takes levothyroxine. She takes no other medications. On examination, her temperature is 99.8°F, heart rate is 102 bpm, blood pressure is 89/62 mm Hg, and normal respiratory rate. She does become light-headed, and her heart rate rises to 125 bpm upon standing with a drop in systolic blood pressure to 70 mm Hg. She is alert and well tanned, with hyperpigmented creases in her hands. Her chest is clear, and her heart rhythm is tachycardic but regular. On abdominal examination, she has normal bowel sounds and mild diffuse tenderness without guarding. Her pulses are rapid and thready. She has no peripheral edema. Initial laboratory studies are significant for Na 121 mEq/L, K 5.8 mEq/L, glucose 52 mg/dL, and creatinine 1.0 mg/dL. what is the most likely diagnosis?

Addison's (addisonian crisis)

•A 63-year-old otherwise healthy man presents with a 3-month history of intermittent "dizziness." He describes the episodes as a sensation of "room spinning" and notes that they are brought on or exacerbated by change in position, particularly when rolling out of bed in the morning. Episodes are generally self-limiting, lasting from 10 to 15 seconds on average. He denies associated nausea, vomiting, blurry or double-vision, hearing loss, tinnitus, ear pain, or headache. He does not take any medications. On examination, he is a well-nourished, healthy-appearing man. Vital signs are within normal limits, and he is afebrile. There are no lesions or masses on his face or head. His voice is normal, and speech is fluent. His ear canals and tympanic membranes are normal-appearing. His symptoms can be provoked by lying down and changing position of his head. While symptomatic, he also develops rotational nystagmus. The nystagmus is self-limiting, with the fast phase occurring counterclockwise (from patient's perspective) as his head is tilted 45-degrees to the left (geotropic nystagmus). He improves a few seconds after sitting up, and nystagmus is no longer noted. The remainder of his general and neurologic examination is unremarkable. what is your diagnosis?

BPPV

ØA 61-year-old woman presents to the emergency room (ER) complaining of cough for 2 weeks. The cough is productive of green sputum and is associated with sweating, shaking chills, and fever up to 102°F (38.8°C). She was exposed to her grandchildren who were told that they had upper respiratory infections 2 weeks ago but now are fine. Her past medical history is significant for diabetes for 10 years, which is under good control using oral hypoglycemics. She denies tobacco, alcohol, or drug use. On examination, she looks ill and in distress, with continuous coughing and chills. Her blood pressure is 100/80 mm Hg, her pulse is 110 beats/min, her temperature is 101°F (38.3°C), her respirations are 24 breaths/min, and her oxygen saturation is 97% on room air. Examination of the head and neck is unremarkable. Her lungs have rales and decreased breath sounds, with dullness to percussion in bilateral bases. Her heart is tachycardic but regular. Her extremities are without signs of cyanosis or edema. The remainder of her examination is normal. A complete blood count (CBC) shows a high white blood cell (WBC) count of 17,000 cells/mm3, with a differential of 85% neutrophils and 20% lymphocytes. Her blood sugar is 120 mg/dL. what is the most likely diagnosis?

CAP

A healthy 52 yo man presents to the doctor's office complaining of increasing fatigue for the past 4 to 5 months. He exercises every day, but lately he has noticed becoming short of breath while jogging. He denies chest pain, orthopnea, paroxysmal nocturnal dyspnea (PND), or swelling in ankles. The patient reports occasional joint pain, for which he uses OTC ibuprofen. He denies bowel changes, melena, or bright red blood per rectum, but he reports vague left sided abdominal pain for a few months off and on, not related to food intake. the patient denies fever, chills, nausea, or vomiting. He lost a few pounds intentionally w/ diet and exercise. On exam, he weighs 205lbs, and he is afebrile. There is slight pallor of the conjunctiva, skin, and palms. No LAD is noted. Chest is clear to auscultation BL. Exam of cards system reveals a RRR w/ no rub or gallop. There is a systolic ejection murmur. His abdomen is soft, nontender, w/o hepatosplenomegaly. Bowel sounds are present. He has no extremity edema, cyanosis, or clubbing. His peripheral pulses are palpable and symmetric. What is your next diagnostic step?

CBC w/ diff. EGD and colonoscopy

A 46-year-old woman presents to your office complaining of a right hand tremor that has been steadily worsening over the past 2 years. She works as a literary agent and states that this tremor is increasingly impairing her ability to work, since this is affecting her dominant hand. She tells you in a slightly quivering voice, "I am often required to take my clients out to lunch, and I get embarrassed when I cannot eat and drink normally. Sometimes, I cannot even drink from a cup without shaking." She finds that a glass of wine with her meal sometimes helps minimize the tremor. On examination, her blood pressure is 125/85 mm Hg, her pulse is 84 beats/min, and her respiratory rate is 16 breaths/min. Neurologic examination reveals a mild head tremor, but no resting tremor of the hands. When she holds a pen by its tip at arm's length however, a coarse bilateral tremor becomes readily visible, which is more pronounced on the right side. The rest of her examination is unremarkable. further evaluation?

CBC, neurologic, med list**, thyroid, EtOH

ØA 47-year-old African-American man presents to your office for a follow-up visit. He was seen 3 weeks ago for an upper respiratory infection and noted incidentally to have a blood pressure of 164/98 mm Hg. He vaguely remembered being told in the past that his blood pressure was "borderline." He feels fine, has no complaints, and his review of systems is entirely negative. He does not smoke cigarettes, drinks "a couple of beers on the weekends," and does not exercise regularly. He has a sedentary job. His father died of a stroke at the age of 69. His mother is alive and in good health at the age of 72. He has two siblings and is not aware of any chronic medical issues that they have. In the office today, his blood pressure is 156/96 mm Hg in his left arm and 152/98 mm Hg in the right arm. He is afebrile, his pulse is 78 beats/min, respiratory rate 14 breaths/min, he is 70-in tall, and weighs 260 lbs. A general physical examination is normal. recommended initial medical management?

CCB (african american male) and ACE

A 52yo man presents to your office for an acute visit because of coughing and SOB. He is well known to you because of multiple office visits in the past few years for similar reasons. He has a chronic "smoker's cough," but reports that in the past 2 days his cough has increased, his sputum has changed from white to green in color, and he has had to increase the frequency with which he uses his albuterol inhaler. He denies having a fever, chest pain, peripheral edema, or other symptoms. His medical hx is significant for HTN, PVD, and two hospitalizations for pneumonia in the past 5 years. He has a 60 pack year hx of smoking and continues to smoke two packs of cigarettes a day. On examination, he is in moderate respiratory distress. His temp is 98.4, BP 152/95, pulse 98 bpm, RR 24 breaths/min, and O2 94% on room air. His lung examination is significant for diffuse expiratory wheezing and a prolonged expiratory phase of respiration. There are no signs of cyanosis. The remainder of his examination is normal. A CXR done in your office shows hyperinflation and a flattened diaphragm, but otherwise he has clear lung fields. What is the most likely cause of this patient's dyspnea?

COPD acute exacerbation

A 58 yo man comes to your office, because of shortness of breath. He has experienced mild dyspnea on exertion for a few years, but more recently he has noted worsening SOB w/ minimal exercise and the onset of dyspnea at rest. He has difficulty reclining; as a result, he spends the night sitting up in a chair trying to sleep. He reports a cough w/ production of yellowish-brown sputum every morning throughout the year. he denies chest pain, ever, chills, or lower extremity edema. He smoked about two packs of cigarettes per day since age 15. He does not drink alcohol. A few months ago, the patient went to an urgent care clinic for evaluation of his symptoms, and he received a prescription for some inhalers, the names of which he does not remember. He was also told to find a primary care physician for further evaluation. On physical exam, his BP is 135/85, pulse 96bpm, resp 28 bpm, and temp 97.6. He is sitting in a chair, leaning forward w/ his arms braced on his knees. He appears uncomfortalbe w/ labored resp and cyanotic lips. He is using accessory muscles of respiration and chest examinatino reveals wheezes and rhonchi BL, but no crackles. the AP of the chest wall appears increased. Cardiovascular examination reveals distant heart sounds but with a RRR. JVD is NL. Extremities w/o cyanosis, edema, or clubbing. what is the most likely diagnosis?

COPD w/ exacerbation, and CHF

ØA 63-year-old African-American woman is brought to the emergency center for upper arm pain and swelling following a fall at home. The family has noted that for approximately the past 2 months, the patient has become progressively fatigued and absentminded, and she has developed loss of appetite and weight loss. She has been getting up to urinate several times per night and complains of thirst; however, a glucose test for diabetes in her doctor's office was negative. This morning, she lost her balance because she felt "light-headed" and fell, landing on her left arm. Physical examination is notable for an elderly, thin woman in mild distress as a result of pain. She is afebrile, and her blood pressure is 110/70 mm Hg and heart rate is 80 bpm. Her thyroid gland is normal to palpation. Her mucous membranes are somewhat dry and sticky. Heart and lung examinations are normal, and carotid auscultation reveals no bruits. Examination of her extremities is significant only for deformity of the left mid-humerus with swelling. The left radial pulse is 2+ and symmetric. The radiologist calls you to confirm the fracture of the mid-left humerus but also states that there is the suggestion of some lytic lesions of the proximal humerus and recommends a skull film. Serum creatinine level is 2.1 mg/dL, with normal electrolyte and glucose concentrations, but serum calcium level is 13 mg/dL and hemoglobin level is 9.2 g/dL. Discuss the criteria for the most likely diagnosis...

CRAB: hyperCalcemia Renal insufficiency Anemia- hgb <12 Bone lesions

ØA 59-year-old woman comes to your clinic because she is concerned that she might have a brain tumor. She has had a fairly severe headache for the last 3 weeks (she rates it as an 8 on a scale of 0-10). She describes the pain as constant, occasionally throbbing but mostly a dull ache, and localized to the right side of her head. She thinks the pain is worse at night, especially when she lies with that side of her head on the pillow. She has had no nausea, vomiting, photophobia, or other visual disturbances. She has had headaches before, but they were mostly occipital and frontal, which she attributed to "stress," and they were relieved with acetaminophen. Her medical history is significant for hypertension, which is controlled with hydrochlorothiazide, and "arthritis" of her neck, shoulders, and hips for which she takes ibuprofen when she feels stiff and achy. On physical examination, her temperature is 100.4°F, heart rate is 88 bpm, blood pressure is 126/75 mm Hg, and respiratory rate is 12 bpm. Her visual acuity is normal, visual fields are intact, and her funduscopic examination is significant for arteriolar narrowing but no papilledema or hemorrhage. She has moderate tenderness over the right side of her head but no obvious scalp lesions. Her chest is clear, and her heart rhythm is regular. Abdominal examination is benign. She has no focal deficits on neurologic examination. She has no joint swelling or deformity but is tender to palpation over her shoulders, hips, and thighs. What is the best next step to confirm the diagnosis?

ESR

A 65yo African-American woman presented to the emergency room complaining of worsening SOB and palpitations for about 1 week. She reports feeling "dizzy" on and off for the past year; the dizziness is associated w/ weakness that has been worsening for the past month. She has been feeling "too tired" to even walk to her backyard and water her flower bed that she used to do "all the time." She has been so dyspneic walking up the stairs at her home that she moved downstairs to the guest room about a week ago. Review of systems is significant is negative. She has no significant medical hx. She had a NL well-woman examination adn screening colonoscopy about 5 years ago. On examination, her bp is 150/85, pulse 98bpm, RR 20 bpm, temp 98.7F, O 99%. Significant findings on examination include conjunctival pallor, mild tenderness w/ deep palpation in the epigastric and left upper quadrant of the abdomen w/ NL bowel sounds, and no organomegaly but positive stool guaiac test. The remainder of the exam, includign resp, cardiovascular, and nervous systems, was normal. What is your next diagnostic step?

Fe

A healthy 52 yo man presents to the doctor's office complaining of increasing fatigue for the past 4 to 5 months. He exercises every day, but lately he has noticed becoming short of breath while jogging. He denies chest pain, orthopnea, paroxysmal nocturnal dyspnea (PND), or swelling in ankles. The patient reports occasional joint pain, for which he uses OTC ibuprofen. He denies bowel changes, melena, or bright red blood per rectum, but he reports vague left sided abdominal pain for a few months off and on, not related to food intake. the patient denies fever, chills, nausea, or vomiting. He lost a few pounds intentionally w/ diet and exercise. On exam, he weighs 205lbs, and he is afebrile. There is slight pallor of the conjunctiva, skin, and palms. No LAD is noted. Chest is clear to auscultation BL. Exam of cards system reveals a RRR w/ no rub or gallop. There is a systolic ejection murmur. His abdomen is soft, nontender, w/o hepatosplenomegaly. Bowel sounds are present. He has no extremity edema, cyanosis, or clubbing. His peripheral pulses are palpable and symmetric. What is the most likely diagnosis?

Fe deficiency anemia

ØYou were busy seeing patients in your outpatient clinic when you heard a commotion coming from the waiting room. You went to check and found a very frantic mother and her 2-year-old son who is clutching his throat, coughing, drooling, and visibly struggling to breathe. The mother endorses that just a few minutes ago, the child was running around while eating grapes when she suddenly heard him gagging and wheezing. Her son has an appointment for well-child examination and he is apparently doing well. He has no significant history of respiratory illness. The toddler is still conscious but unable to talk, and his cough is becoming weaker. Breath sounds are decreased bilaterally, with wheezing and stridor heard on auscultation. You tried to ventilate the patient with the chin-lift maneuver but the chest fails to rise. You opened the mouth but you are unable to see any foreign object. what is the most likely diagnosis?

Foreign Body Airway Obstruction

•A 45-year-old man presents to the clinic with a cough productive of purulent sputum of 3 weeks' duration. He says that he had just gotten over a cold a few weeks prior to this episode. He occasionally has fevers, and he coughs so much that he has chest pain. He reports having a mild sore throat and nasal congestion. He has no history of asthma or any chronic lung diseases. He denies nausea, vomiting, diarrhea, and any recent travel. He denies any smoking history. On examination, his temperature is 98.6 °F (37.0 °C), pulse is 96 beats/min, blood pressure is 124/82 mm Hg, respiratory rate is 18 breaths/min, and oxygen saturation is 99% on room air. Head, ears, eyes, nose, and throat (HEENT) examination reveals no erythema of the posterior oropharynx, tonsillar exudates, uvular deviations, or significant tonsillar swelling. Neck examination is negative. The chest examination yields occasional wheezes, but normal air movement is noted. what are some common noninfectious causes of cough?

GERD, Ca., ACE inhibitor, CHF, asthma/COPD, noxious stimuli, allergic rhinitis w/ postnasal drip

ØAn 18-month-old girl is brought to the office by her mother because of a rash. The patient has had a subjective high fever for the past 3 days, along with some mild respiratory symptoms of cough and rhinorrhea. She was given acetaminophen for the fever but no other medications. The fever has gone down in the past day, but today she developed an erythematous rash that developed suddenly, starting on the trunk and spreading to the extremities. The child has no significant medical history and no known sick contacts, although she attends day care 3 days a week. On examination, she is mildly fussy but is easily consolable in her mother's lap. Upon inspection, the rash consists of small macules and papules that blanch on palpation. The remainder of her examination is unremarkable. What is the most likely cause of the illness?

HHV6

ØA 47-year-old African-American man presents to your office for a follow-up visit. He was seen 3 weeks ago for an upper respiratory infection and noted incidentally to have a blood pressure of 164/98 mm Hg. He vaguely remembered being told in the past that his blood pressure was "borderline." He feels fine, has no complaints, and his review of systems is entirely negative. He does not smoke cigarettes, drinks "a couple of beers on the weekends," and does not exercise regularly. He has a sedentary job. His father died of a stroke at the age of 69. His mother is alive and in good health at the age of 72. He has two siblings and is not aware of any chronic medical issues that they have. In the office today, his blood pressure is 156/96 mm Hg in his left arm and 152/98 mm Hg in the right arm. He is afebrile, his pulse is 78 beats/min, respiratory rate 14 breaths/min, he is 70-in tall, and weighs 260 lbs. A general physical examination is normal. what is his dx?

HTN and obesity

A 48-yo man comes to your office for a routine physical examination. He is a new patient to your practice. He has no significant medical history and takes no medications regularly. His father died at the age of 74 of a heart attack. His mother is alive at the age of 80. She has HTN. He has two younger siblings w/ no known chronic medical conditions. He does not smoke cigarettes, drink alcohol, or use any recreational drugs. He does not exercise. On examination, his BP is 127/82, pulse 80 bpm, respiratory rate 18 breaths/min, ht 67in, wt 90lb. On careful physical exam, no abnormalities are noted. What screening tests for cardiovascular disease should be recommended for this patient?

HTN, dyslipidemia

A 35-year-old woman with a history of asthma presents to your office with symptoms of nasal itching, sneezing, and rhinorrhea. She states she feels this way most days, but her symptoms are worse in the spring and fall. She has had difficulty sleeping because she is always congested. She states she has taken diphenhydramine (Benadryl) with no relief. She does not smoke cigarettes and does not have exposure to passive smoke, but she does have two cats at home. On examination, she appears tired but is in no respiratory distress. Her vital signs are as follows: temperature 98.8 °F, blood pressure 128/84 mm Hg, pulse 88 beats/min, and respiratory rate 18 breaths/min. The mucosa of her nasal turbinates appears swollen (boggy) and has a pale, bluish-gray color. Thin and watery secretions are seen. No abnormalities are seen on ear examination. There is no cervical lymphadenopathy noted, and her lungs are clear. what is your next step in management?

IN CCS (fluticasone), decongestants or antihistamines for short term, allergy testing, zyrtec

ØA 26-year-old woman presents to the emergency center on a Saturday afternoon complaining of bleeding from her nose and mouth since the previous night. She also noticed small, reddish spots on her lower extremities when she got out of the bed this morning. She denies fever, chills, nausea, vomiting, abdominal pain, or joint pain. The patient reports she had developed an upper respiratory infection 2 weeks prior to the emergency room visit, but the infection has now resolved. She denies significant medical problems. Her menses have been normal, and her last menstrual period was approximately 2 weeks ago. She denies excessive bleeding in the past, even after delivering her baby. Prior to this episode, she never had epistaxis, easy bruisability, or bleeding into her joints. There is no family history of abnormal bleeding. The patient does not take any medications. On examination she is alert, oriented, and somewhat anxious. Her blood pressure is 110/70 mm Hg, her heart rate is 90 bpm, and she is afebrile. No pallor or jaundice is noted. There is bright red oozing from the nose and the gingiva. Skin examination reveals multiple 1-mm flat reddish spots on her lower extremities. The rest of the examination is normal. There is no lymphadenopathy or hepatosplenomegaly. Her complete blood cell count (CBC) is normal except for a platelet count of 18,000/mm3. Prothrombin time (PT) and partial thromboplastin time (PTT) are normal. what is the most likely diagnosis?

ITP (immune thrombocytopenic purpura)

ØA 26-year-old woman presents to the emergency center on a Saturday afternoon complaining of bleeding from her nose and mouth since the previous night. She also noticed small, reddish spots on her lower extremities when she got out of bed this morning. She denies fever, chills, nausea, vomiting, abdominal pain, or joint pain. The patient reports she had developed an upper respiratory infection 2 weeks prior to the emergency room visit, but the infection has now resolved. She denies significant medical problems. Her menses have been normal, and her last menstrual period was approximately 2 weeks ago. She denies excessive bleeding in the past, even after delivering her baby. Prior to this episode, she never had epistaxis, easy bruisability, or bleeding into her joints. There is no family history of abnormal bleeding. The patient does not take any medications. On examination she is alert, oriented, and somewhat anxious. Her blood pressure is 110/70 mm Hg, her heart rate is 90 bpm, and she is afebrile. No pallor or jaundice is noted. There is bright red oozing from the nose and the gingiva. Skin examination reveals multiple 1-mm flat reddish spots on her lower extremities. The rest of the examination is normal. There is no lymphadenopathy or hepatosplenomegaly. Her complete blood cell count (CBC) is normal except for a platelet count of 18,000/mm3. Prothrombin time (PT) and partial thromboplastin time (PTT) are normal. what is the best initial treatment?

IVIG and PO steroids, tx underlying infxn, platelet

ØAn 83-year-old woman is brought to the clinic by her husband who was concerned with his wife's memory problems. He first noticed some memory decline a few years ago, but the onset was subtle and did not interfere with her day-to-day activities. Mainly, she has some difficulty remembering details, is repeating things, and is being forgetful. ØThe patient's family noticed her gradually increasing memory problems, particularly over the past year. She is unable to remember her appointments and relies heavily on written notes and appointment books. Recently, she got lost while driving and was found by her family 10 hours later. She was unable to use her cell phone and was unsure about her home address and phone number. The patient says that she has always been forgetful. ØHer medical history is significant for well-controlled hypertension and a history of mastectomy secondary to breast cancer diagnosed 20 years ago. She has no significant history of tobacco or alcohol use. ØShe is independent with all activities of daily living, but needs assistance with medication administration, banking, and transportation. She is up-to-date with her health maintenance and immunization. what office testing to determine dx?

MMSE, MOCA, depression screen

A 55yo caucasian woman, new to your practice, presents for an "annual physical examination." She reports that she is healthy, generally feels well, and has no specific complaints. She has a history of having had a "total hysterectomy," by which she means that her uterus, cervix, ovaries, and fallopian tubes were removed. The surgery was performed because of fibroids. She has had a Pap smear every 3 years since the age of 21, all of which have been normal. She has no other significant medical or surgical history. She takes a multivitamin pill daily but no other medications. Her family history is significant for breast cancer that was diagnosed in her maternal grandmother at the age of 72. The patient is married, monogamous, and does not smoke cigarettes or drink alcohol. She tries to avoid dairy products because of "lactose intolerance." She walks 30min a day five times a week for exercise. Her physical examination is normal. For this patient, how often should a pap smear be performed?

N/a, her Total hysterectomy

A 37-year-old previously healthy woman presents to your clinic for unintentional weight loss. Over the past 3 months, she has lost approximately 15 lb without changing her diet or activity level. Otherwise, she feels great. She has an excellent appetite, no gastrointestinal complaints except for occasional loose stools, a good energy level, and no complaints of fatigue. She denies heat or cold intolerance. On examination, her heart rate is 108 bpm, blood pressure 142/82 mm Hg, and she is afebrile. When she looks at you, she seems to stare, and her eyes are somewhat protuberant. You note a large, smooth, nontender thyroid gland and a 2/6 systolic ejection murmur on cardiac examination, and her skin is warm and dry. There is a fine resting tremor. How might treatment change if she were pregnant?

NOT methimazole or RI

A 52yo man presents to your office for an acute visit because of coughing and SOB. He is well known to you because of multiple office visits in the past few years for similar reasons. He has a chronic "smoker's cough," but reports that in the past 2 days his cough has increased, his sputum has changed from white to green in color, and he has had to increase the frequency with which he uses his albuterol inhaler. He denies having a fever, chest pain, peripheral edema, or other symptoms. His medical hx is significant for HTN, PVD, and two hospitalizations for pneumonia in the past 5 years. He has a 60 pack year hx of smoking and continues to smoke two packs of cigarettes a day. On examination, he is in moderate respiratory distress. His temp is 98.4, BP 152/95, pulse 98 bpm, RR 24 breaths/min, and O2 94% on room air. His lung examination is significant for diffuse expiratory wheezing and a prolonged expiratory phase of respiration. There are no signs of cyanosis. The remainder of his examination is normal. A CXR done in your office shows hyperinflation and a flattened diaphragm, but otherwise he has clear lung fields. What acute treatments are most appropriate at this time?

O2 (maybe), beta agonist, steroid (systemic), antibiotic (b/c change in sputum)

ØA 48-year-old woman calls 911 and is brought to the emergency center complaining of a sudden onset of dyspnea. She reports she was standing in the kitchen making dinner, when she suddenly felt as if she could not get enough air, her heart started racing, and she became light-headed and felt as if she would faint. She denied chest pain or cough. Her medical history is significant only for gallstones, for which she underwent a cholecystectomy 2 weeks previously. The procedure was complicated by a wound infection, requiring her to stay in the hospital for 8 days. She takes no medications regularly, and only takes acetaminophen as needed for pain at her abdominal incision site. ØOn examination, she is tachypneic with a respiratory rate of 28 bpm, oxygen saturation 84% on room air, heart rate 124 bpm, and blood pressure 118/89 mm Hg. She appears uncomfortable, diaphoretic, and frightened. Her oral mucosa is slightly cyanotic, her jugular venous pressure is elevated, and her chest is clear to auscultation. Her heart rhythm is tachycardic but regular with a loud second sound in the pulmonic area, but no gallop or murmur. Her abdominal examination is benign, with a clean incision site without signs of infection. Her right leg is moderately swollen from mid-thigh to her feet, and her thigh and calf are mildly tender to palpation. Laboratory studies including cardiac enzymes are normal; her electrocardiogram (ECG) reveals only sinus tachycardia, and her chest x-ray is interpreted as normal. what is the most likely diagnosis?

PE

ØA 59-year-old woman comes to your clinic because she is concerned that she might have a brain tumor. She has had a fairly severe headache for the last 3 weeks (she rates it as an 8 on a scale of 0-10). She describes the pain as constant, occasionally throbbing but mostly a dull ache, and localized to the right side of her head. She thinks the pain is worse at night, especially when she lies with that side of her head on the pillow. She has had no nausea, vomiting, photophobia, or other visual disturbances. She has had headaches before, but they were mostly occipital and frontal, which she attributed to "stress," and they were relieved with acetaminophen. Her medical history is significant for hypertension, which is controlled with hydrochlorothiazide, and "arthritis" of her neck, shoulders, and hips for which she takes ibuprofen when she feels stiff and achy. On physical examination, her temperature is 100.4°F, heart rate is 88 bpm, blood pressure is 126/75 mm Hg, and respiratory rate is 12 bpm. Her visual acuity is normal, visual fields are intact, and her funduscopic examination is significant for arteriolar narrowing but no papilledema or hemorrhage. She has moderate tenderness over the right side of her head but no obvious scalp lesions. Her chest is clear, and her heart rhythm is regular. Abdominal examination is benign. She has no focal deficits on neurologic examination. She has no joint swelling or deformity but is tender to palpation over her shoulders, hips, and thighs. What is the most likely diagnosis?

PMR and GCA

A 44-year-old man presents with sudden onset of shaking chills, fever, and productive cough. He was in his usual state of good health until 1 week ago, when he developed mild nasal congestion and achiness. He otherwise felt well until last night, when he became fatigued and feverish, and developed a cough associated with right-sided pleuritic chest pain. His medical history is remarkable only for his 15-pack per year smoking habit. In your office, his vital signs are normal except for a temperature of 102°F. His oxygen saturation on room air is 100%. He is comfortable, except when he coughs. His physical examination is unremarkable except for bronchial breath sounds and end-inspiratory crackles in the right lower lung field. There is a right lower lobe consolidation on chest x-ray. what is the diagnosis?

PNA- lobar bacterial CAP

ØA 32-year-old woman presents to the emergency center complaining of productive cough, fever, and chest pain for 4 days. She was seen 2 days ago in her primary care physician's clinic with the same complaints, was diagnosed clinically with pneumonia, and was sent home with oral azithromycin. Since then, her cough has diminished in quantity. However, the fever has not abated, and she still experiences left-sided chest pain, which is worse when she coughs or takes a deep breath. In addition, she has started to feel short of breath when she walks around the house. She has no other medical history. She does not smoke and has no history of occupational exposure. She has not traveled outside of the United States and has no sick contacts. ØOn physical examination, her temperature is 103.4°F, heart rate is 116 bpm, blood pressure is 128/69 mm Hg, respiratory rate is 24 bpm and is shallow. Her pulse oximetry is 94% saturation on room air. Physical examination is significant for decreased breath sounds in the lower half of the left lung fields posteriorly, with dullness to percussion about halfway up. There are a few inspiratory crackles in the mid-lung fields, and her right side is clear to auscultation. Her heart is tachycardic but regular with no murmurs. She has no cyanosis.

Pleural effusion (parapneumonic)

ØA 59-year-old woman comes to your clinic because she is concerned that she might have a brain tumor. She has had a fairly severe headache for the last 3 weeks (she rates it as an 8 on a scale of 0-10). She describes the pain as constant, occasionally throbbing but mostly a dull ache, and localized to the right side of her head. She thinks the pain is worse at night, especially when she lies with that side of her head on the pillow. She has had no nausea, vomiting, photophobia, or other visual disturbances. She has had headaches before, but they were mostly occipital and frontal, which she attributed to "stress," and they were relieved with acetaminophen. Her medical history is significant for hypertension, which is controlled with hydrochlorothiazide, and "arthritis" of her neck, shoulders, and hips for which she takes ibuprofen when she feels stiff and achy. On physical examination, her temperature is 100.4°F, heart rate is 88 bpm, blood pressure is 126/75 mm Hg, and respiratory rate is 12 bpm. Her visual acuity is normal, visual fields are intact, and her funduscopic examination is significant for arteriolar narrowing but no papilledema or hemorrhage. She has moderate tenderness over the right side of her head but no obvious scalp lesions. Her chest is clear, and her heart rhythm is regular. Abdominal examination is benign. She has no focal deficits on neurologic examination. She has no joint swelling or deformity but is tender to palpation over her shoulders, hips, and thighs. What is the treatment for the most likely diagnosis?

Prednisone (roids, shoot em up!, used to prevent blindness)

A 55yo man comes into your office for f/u of a chronic cough. He also complains of SOB w/ activity. He reports that this has been getting worse over time. As you are interviewing the pt, you note that he smells of cigarette smoke. Upon further question, he reports smoking 1 pack of cigarettes per day for the past 35 years and denies ever being advised to quit. On exam, he is in no resp distress at rest, VS are NL, and no obvious cyanosis. His pulmonary examination is notable for reduced air movement and faint expiratory wheezing on auscultation. What would you recommend to this patient?

QUIT SMOKING

•A 42-year-old Hispanic woman presents to the emergency department (ED) complaining of 24 hours of severe, steady epigastric abdominal pain, radiating to her back, with several episodes of nausea and vomiting. She has experienced similar painful episodes in the past, usually in the evening following heavy meals, but the episodes always resolved spontaneously within an hour or two. This time the pain did not improve, so she sought medical attention. No medical history and no medications. She is married, has three children, and does not drink alcohol or smoke cigarettes. •On examination, she is afebrile, tachycardic with a heart rate of 104 bpm, blood pressure of 115/74 mm Hg, and shallow respirations of 22 bpm. She is moving uncomfortably on the stretcher, her skin is warm and diaphoretic, and she has scleral icterus. Her abdomen is soft, mildly distended with marked right upper quadrant and epigastric tenderness to palpation, hypoactive bowel sounds, and no masses or organomegaly appreciated. Her stool is negative for occult blood. Laboratory studies are significant for a total bilirubin (9.2 g/dL) with a direct fraction of 4.8 g/dL, alkaline phosphatase 285 IU/L, aspartate aminotransferase (AST) 78 IU/L, alanine aminotransferase (ALT) 92 IU/L, and elevated amylase level 1249 IU/L. Her leukocyte count is 16,500/mm3 with 82% polymorphonuclear cells and 16% lymphocytes. Serum electrolytes, blood urea nitrogen (BUN), and creatinine are normal. A plain film of the abdomen shows a nonspecific gas pattern and no pneumoperitoneum, and chest x-ray is normal. what is the next dx step?

RUQ U/S and CMP/CBC

ØAn 18-month-old girl is brought to the office by her mother because of a rash. The patient has had a subjective high fever for the past 3 days, along with some mild respiratory symptoms of cough and rhinorrhea. She was given acetaminophen for the fever but no other medications. The fever has gone down in the past day, but today she developed an erythematous rash that developed suddenly, starting on the trunk and spreading to the extremities. The child has no significant medical history and no known sick contacts, although she attends day care 3 days a week. On examination, she is mildly fussy but is easily consolable in her mother's lap. Upon inspection, the rash consists of small macules and papules that blanch on palpation. The remainder of her examination is unremarkable. what is the most likely dx?

Roseola

A 52yo man presents to your office for an acute visit because of coughing and SOB. He is well known to you because of multiple office visits in the past few years for similar reasons. He has a chronic "smoker's cough," but reports that in the past 2 days his cough has increased, his sputum has changed from white to green in color, and he has had to increase the frequency with which he uses his albuterol inhaler. He denies having a fever, chest pain, peripheral edema, or other symptoms. His medical hx is significant for HTN, PVD, and two hospitalizations for pneumonia in the past 5 years. He has a 60 pack year hx of smoking and continues to smoke two packs of cigarettes a day. On examination, he is in moderate respiratory distress. His temp is 98.4, BP 152/95, pulse 98 bpm, RR 24 breaths/min, and O2 94% on room air. His lung examination is significant for diffuse expiratory wheezing and a prolonged expiratory phase of respiration. There are no signs of cyanosis. The remainder of his examination is normal. A CXR done in your office shows hyperinflation and a flattened diaphragm, but otherwise he has clear lung fields. What interventions would be most helpful to reduce risk of future exacerbations of this condition?

Sama/Saba, Lama/Laba, pneumoccoccal vaccines, STOP SMOKING

A 72-year-old man is admitted to the hospital because of the acute onset of a right facial droop, right arm weakness, and some difficulty speaking. These symptoms started 6 hours ago while he was sitting at the breakfast table. He had no headache, no diminishment of consciousness, and no abnormal involuntary movements. Two weeks ago, he had a transient painless loss of vision in his left eye, which resolved spontaneously within a few hours. His medical history is significant for long-standing hypertension and a myocardial infarction (MI) 4 years previously, which was treated with percutaneous angioplasty. His medications include a daily aspirin, metoprolol, and simvastatin. He does not smoke. When you see him in the emergency room, his symptoms have nearly resolved. ØHe is afebrile, heart rate is 62 bpm, and blood pressure is 135/87 mm Hg. The corner of his mouth droops, with slight flattening of the right nasolabial fold, but he is able to fully elevate his eyebrows. His strength is 4/5 in his right arm and hand, and the rest of his neurologic examination is normal. He has no carotid bruits, his heart rhythm is regular with no murmur but with an S4 gallop. The remainder of his physical examination is normal. Laboratory studies, including renal function, liver function, lipid profile, and complete blood count (CBC), all are normal. Within a few hours, all of the patient's symptoms have resolved. ØWhat is the likely diagnosis and likely CAUSE (anatomical location)?

TIA of possibly internal carotid

•A 40-year-old man presents to the clinic complaining of having 10 episodes of watery, nonbloody diarrhea that started last night. He vomited twice last night but has been able to tolerate liquids today. He has had intermittent abdominal cramps as well. He reports having muscle aches, weakness, headache, and low-grade temperature. He is here with his daughter, who started with the same symptoms this morning. On questioning, he states that he has no significant medical history, no surgeries, and does not take any medications. He does not smoke cigarettes, drink alcohol, use any illicit drugs, and has never had a blood transfusion. He and his family returned to the United States yesterday, following a week-long vacation in Mexico. On examination, he is not in acute distress. His blood pressure is 110/60 mm Hg, his pulse is 98 beats/min, his respiratory rate is 16 breaths/min, and his temperature is 99.1°F (37.2°C). His mucous membranes are dry. His bowel sounds are hyperactive and his abdomen is mildly tender throughout, but there is no rebound tenderness and no guarding. A rectal examination is normal and his stool is guaiac negative. The remainder of his examination is unremarkable. what is the most likely dx?

Traveler's Diarrhea (bacterial gastroenteritis) enterotoxigenic E.coli

ØA 63-year-old African-American woman is brought to the emergency center for upper arm pain and swelling following a fall at home. The family has noted that for approximately the past 2 months, the patient has become progressively fatigued and absentminded, and she has developed loss of appetite and weight loss. She has been getting up to urinate several times per night and complains of thirst; however, a glucose test for diabetes in her doctor's office was negative. This morning, she lost her balance because she felt "light-headed" and fell, landing on her left arm. Physical examination is notable for an elderly, thin woman in mild distress as a result of pain. She is afebrile, and her blood pressure is 110/70 mm Hg and heart rate is 80 bpm. Her thyroid gland is normal to palpation. Her mucous membranes are somewhat dry and sticky. Heart and lung examinations are normal, and carotid auscultation reveals no bruits. Examination of her extremities is significant only for deformity of the left mid-humerus with swelling. The left radial pulse is 2+ and symmetric. The radiologist calls you to confirm the fracture of the mid-left humerus but also states that there is the suggestion of some lytic lesions of the proximal humerus and recommends a skull film. Serum creatinine level is 2.1 mg/dL, with normal electrolyte and glucose concentrations, but serum calcium level is 13 mg/dL and hemoglobin level is 9.2 g/dL. What is your next therapeutic step?

U/A, splint, SPEP, IVF for hypercalcemia, refer to onc and ortho

ØA 48-year-old woman calls 911 and is brought to the emergency center complaining of a sudden onset of dyspnea. She reports she was standing in the kitchen making dinner, when she suddenly felt as if she could not get enough air, her heart started racing, and she became light-headed and felt as if she would faint. She denied chest pain or cough. Her medical history is significant only for gallstones, for which she underwent a cholecystectomy 2 weeks previously. The procedure was complicated by a wound infection, requiring her to stay in the hospital for 8 days. She takes no medications regularly, and only takes acetaminophen as needed for pain at her abdominal incision site. ØOn examination, she is tachypneic with a respiratory rate of 28 bpm, oxygen saturation 84% on room air, heart rate 124 bpm, and blood pressure 118/89 mm Hg. She appears uncomfortable, diaphoretic, and frightened. Her oral mucosa is slightly cyanotic, her jugular venous pressure is elevated, and her chest is clear to auscultation. Her heart rhythm is tachycardic but regular with a loud second sound in the pulmonic area, but no gallop or murmur. Her abdominal examination is benign, with a clean incision site without signs of infection. Her right leg is moderately swollen from mid-thigh to her feet, and her thigh and calf are mildly tender to palpation. Laboratory studies including cardiac enzymes are normal; her electrocardiogram (ECG) reveals only sinus tachycardia, and her chest x-ray is interpreted as normal. What are the common risk factors for this condition?

Virchows: age, sx, immobility, hx DVT

A 44-year-old man presents with sudden onset of shaking chills, fever, and productive cough. He was in his usual state of good health until 1 week ago, when he developed mild nasal congestion and achiness. He otherwise felt well until last night, when he became fatigued and feverish, and developed a cough associated with right-sided pleuritic chest pain. His medical history is remarkable only for his 15-pack per year smoking habit. In your office, his vital signs are normal except for a temperature of 102°F. His oxygen saturation on room air is 100%. He is comfortable, except when he coughs. His physical examination is unremarkable except for bronchial breath sounds and end-inspiratory crackles in the right lower lung field. There is a right lower lobe consolidation on chest x-ray. what is the next step (tx in/outpt)?

abx, antipyretics, cough suppressants, close out pt w/ f/u 1-2wks

•A 45-year-old man presents to the clinic with a cough productive of purulent sputum of 3 weeks' duration. He says that he had just gotten over a cold a few weeks prior to this episode. He occasionally has fevers, and he coughs so much that he has chest pain. He reports having a mild sore throat and nasal congestion. He has no history of asthma or any chronic lung diseases. He denies nausea, vomiting, diarrhea, and any recent travel. He denies any smoking history. On examination, his temperature is 98.6 °F (37.0 °C), pulse is 96 beats/min, blood pressure is 124/82 mm Hg, respiratory rate is 18 breaths/min, and oxygen saturation is 99% on room air. Head, ears, eyes, nose, and throat (HEENT) examination reveals no erythema of the posterior oropharynx, tonsillar exudates, uvular deviations, or significant tonsillar swelling. Neck examination is negative. The chest examination yields occasional wheezes, but normal air movement is noted. What is the most likely diagnosis?

acute bronchitis

ØA 25-year-old African-American man is admitted to your service with the diagnosis of a sickle cell pain episode. He was admitted to the hospital six times last year with the same diagnosis, and he was last discharged 2 months ago. This time, he presented to the emergency department complaining of abdominal and bilateral lower extremity pain, his usual sites of pain. When you examine him, you note he is febrile to 101°F, with respiratory rate 25 bpm, normal blood pressure, HR 100 bpm. Lung examination reveals bronchial breath sounds and egophony in the right lung base. His oxygen saturation on 2 L/min nasal cannula is 92%. Besides the usual abdominal and leg pain, he is now complaining of chest pain, which is worse on inspiration. Although he is tender on palpation of his extremities, the remainder of his examination is normal. His laboratory examinations reveal elevated white blood cell and reticulocyte counts, and a hemoglobin and hematocrit that are slightly lower than baseline. The peripheral smear is shown. what is the most likely dx?

acute chest syndrome due to sickle cell anemia

ØA previously healthy 48-year-old accountant presents to his primary care office with severe low back pain that began the previous day after he helped his daughter move into her college dorm. He denies any trauma or previous back injury. He describes the pain as generally "achy," and sometimes characterized as being "sharp" when he moves suddenly. The pain is located in his lower back and radiates down the back of both legs to the middle of his posterior thighs. He has been continent of both bowel and bladder and denies any weakness in his legs. He denies fever, chills, weight loss, or malaise. He finds it very difficult to stand for prolonged periods of time and it has not been relieved with acetaminophen or ibuprofen. His past medical history is significant for hypertension; his only medications are metoprolol and a baby aspirin daily. He does not smoke or use illicit drugs, and only drinks alcohol on occasion. On physical examination, he is well-developed, overweight, and in moderate discomfort. On neuromuscular examination, he has moderate tenderness bilaterally in his lumbar paraspinous muscles, and his lumbar flexion and extension are limited by pain. Strength and sensation are within normal limits and equal and symmetrical bilaterally, as are his deep tendon knee and ankle reflexes. Straight leg raise testing is negative bilaterally and gait is within normal limits. what is the most likely dx?

acute low back pain, lumbosacral strain

ØA 4-year-old boy is seen by his pediatrician for easy bruising, joint pain, and leg pain; red dots on the skin that do not blanch; and hepatosplenomegaly. The complete blood count (CBC) reveals an elevated white blood cell count (50,000/mm3), a low hemoglobin level (anemia), and thrombocytopenia (low platelet count). Examination of the peripheral smear of the blood shows numerous cells with a high nuclear to cytoplasmic ratio, and fine chromatin; the complete blood count shows anemia and thrombocytopenia. what is the most likely dx?

acute lymphoblastic leukemia

•A 42-year-old Hispanic woman presents to the emergency department (ED) complaining of 24 hours of severe, steady epigastric abdominal pain, radiating to her back, with several episodes of nausea and vomiting. She has experienced similar painful episodes in the past, usually in the evening following heavy meals, but the episodes always resolved spontaneously within an hour or two. This time the pain did not improve, so she sought medical attention. No medical history and no medications. She is married, has three children, and does not drink alcohol or smoke cigarettes. •On examination, she is afebrile, tachycardic with a heart rate of 104 bpm, blood pressure of 115/74 mm Hg, and shallow respirations of 22 bpm. She is moving uncomfortably on the stretcher, her skin is warm and diaphoretic, and she has scleral icterus. Her abdomen is soft, mildly distended with marked right upper quadrant and epigastric tenderness to palpation, hypoactive bowel sounds, and no masses or organomegaly appreciated. Her stool is negative for occult blood. Laboratory studies are significant for a total bilirubin (9.2 g/dL) with a direct fraction of 4.8 g/dL, alkaline phosphatase 285 IU/L, aspartate aminotransferase (AST) 78 IU/L, alanine aminotransferase (ALT) 92 IU/L, and elevated amylase level 1249 IU/L. Her leukocyte count is 16,500/mm3 with 82% polymorphonuclear cells and 16% lymphocytes. Serum electrolytes, blood urea nitrogen (BUN), and creatinine are normal. A plain film of the abdomen shows a nonspecific gas pattern and no pneumoperitoneum, and chest x-ray is normal. what is the most likely dx?

acute pancreatitis

A 35-year-old woman with a history of asthma presents to your office with symptoms of nasal itching, sneezing, and rhinorrhea. She states she feels this way most days, but her symptoms are worse in the spring and fall. She has had difficulty sleeping because she is always congested. She states she has taken diphenhydramine (Benadryl) with no relief. She does not smoke cigarettes and does not have exposure to passive smoke, but she does have two cats at home. On examination, she appears tired but is in no respiratory distress. Her vital signs are as follows: temperature 98.8 °F, blood pressure 128/84 mm Hg, pulse 88 beats/min, and respiratory rate 18 breaths/min. The mucosa of her nasal turbinates appears swollen (boggy) and has a pale, bluish-gray color. Thin and watery secretions are seen. No abnormalities are seen on ear examination. There is no cervical lymphadenopathy noted, and her lungs are clear. what is the most likely diagnosis?

allergic rhinitis

•A 4-year-old child is seen by the pediatrician for ear pain. His mother reports he was in his usual state of good health until about 5 days ago when he developed an upper respiratory infection (URI) consisting of clear nasal discharge and cough. He had been having normal activity and intake until about 48 hours prior when he developed a temperature of 102.1°F (38.9°C) and complaints that his left ear hurts. She denies nausea, vomiting, diarrhea, headache, or change in urine output. She reports that he did not sleep well the previous evening, awakening several times complaining of ear pain, but remains somewhat interested in his toys earlier in the day. what is the best treatment?

amoxicillin, augmentin, ceftriaxone

A 65yo African-American woman presented to the emergency room complaining of worsening SOB and palpitations for about 1 week. She reports feeling "dizzy" on and off for the past year; the dizziness is associated w/ weakness that has been worsening for the past month. She has been feeling "too tired" to even walk to her backyard and water her flower bed that she used to do "all the time." She has been so dyspneic walking up the stairs at her home that she moved downstairs to the guest room about a week ago. Review of systems is significant is negative. She has no significant medical hx. She had a NL well-woman examination adn screening colonoscopy about 5 years ago. On examination, her bp is 150/85, pulse 98bpm, RR 20 bpm, temp 98.7F, O 99%. Significant findings on examination include conjunctival pallor, mild tenderness w/ deep palpation in the epigastric and left upper quadrant of the abdomen w/ NL bowel sounds, and no organomegaly but positive stool guaiac test. The remainder of the exam, including resp, cardiovascular, and nervous systems, was normal. What is the most likely diagnosis?

anemia w/ lower GI bleed

ØA 37-year-old man presents to your office with the complaint of cough. The cough began approximately 3 months prior to this appointment, and it has become more annoying to the patient. The cough is nonproductive and worse at night and after exercise. He has had a sedentary lifestyle but recently started an exercise program, including jogging, and says he is having a much harder time with the exertion. He runs out of breath earlier than he did to previously, and he coughs a lot. He has not had any fever, blood-tinged sputum, or weight loss. He denies nasal congestion and headaches. He does not smoke and has no significant medical history. His examination is notable for a blood pressure of 134/78 mm Hg and lung findings of occasional expiratory wheezes on forced expiration. A chest radiograph is read as normal. what is the most likely diagnosis?

asthma

ØA 50-year-old male with a past medical history of hypertension and a history of palpitations presents for follow-up of his hypertension. He is adherent with a low sodium diet, exercises 150 minutes weekly, and is taking his metoprolol at maximum dose (for the HTN and palpitations). His home blood pressure logs and the clinic reading reveal blood pressures in the range 140-150/90-100. The rest of his vitals including heart rate are normal, as his examination. You decide to add a thiazide diuretic to his existing antihypertensive regimen. what is the mechanism of action of metoprolol?

beta 1 selective adrenoreceptor antagonist

A 55yo caucasian woman, new to your practice, presents for an "annual physical examination." She reports that she is healthy, generally feels well, and has no specific complaints. She has a history of having had a "total hysterectomy," by which she means that her uterus, cervix, ovaries, and fallopian tubes were removed. The surgery was performed because of fibroids. She has had a Pap smear every 3 years since the age of 21, all of which have been normal. She has no other significant medical or surgical history. She takes a multivitamin pill daily but no other medications. Her family history is significant for breast cancer that was diagnosed in her maternal grandmother at the age of 72. The patient is married, monogamous, and does not smoke cigarettes or drink alcohol. She tries to avoid dairy products because of "lactose intolerance." She walks 30min a day five times a week for exercise. Her physical examination is normal. According to USPSTF, what is the recommended interval for screening mammography?

biennially 50-74

ØA 4-year-old boy is seen by his pediatrician for easy bruising, joint pain, and leg pain; red dots on the skin that do not blanch; and hepatosplenomegaly. The complete blood count (CBC) reveals an elevated white blood cell count (50,000/mm3), a low hemoglobin level (anemia), and thrombocytopenia (low platelet count). Examination of the peripheral smear of the blood shows numerous cells with a high nuclear to cytoplasmic ratio, and fine chromatin; the complete blood count shows anemia and thrombocytopenia. what additional studies?

bone marrow bx

•A 63-year-old otherwise healthy man presents with a 3-month history of intermittent "dizziness." He describes the episodes as a sensation of "room spinning" and notes that they are brought on or exacerbated by change in position, particularly when rolling out of bed in the morning. Episodes are generally self-limiting, lasting from 10 to 15 seconds on average. He denies associated nausea, vomiting, blurry or double-vision, hearing loss, tinnitus, ear pain, or headache. He does not take any medications. On examination, he is a well-nourished, healthy-appearing man. Vital signs are within normal limits, and he is afebrile. There are no lesions or masses on his face or head. His voice is normal, and speech is fluent. His ear canals and tympanic membranes are normal-appearing. His symptoms can be provoked by lying down and changing position of his head. While symptomatic, he also develops rotational nystagmus. The nystagmus is self-limiting, with the fast phase occurring counterclockwise (from patient's perspective) as his head is tilted 45-degrees to the left (geotropic nystagmus). He improves a few seconds after sitting up, and nystagmus is no longer noted. The remainder of his general and neurologic examination is unremarkable. what is the next therapeutic step?

canalith repositioning maneuvers or vestibular rehabilitation

•A 42-year-old Hispanic woman presents to the emergency department (ED) complaining of 24 hours of severe, steady epigastric abdominal pain, radiating to her back, with several episodes of nausea and vomiting. She has experienced similar painful episodes in the past, usually in the evening following heavy meals, but the episodes always resolved spontaneously within an hour or two. This time the pain did not improve, so she sought medical attention. No medical history and no medications. She is married, has three children, and does not drink alcohol or smoke cigarettes. •On examination, she is afebrile, tachycardic with a heart rate of 104 bpm, blood pressure of 115/74 mm Hg, and shallow respirations of 22 bpm. She is moving uncomfortably on the stretcher, her skin is warm and diaphoretic, and she has scleral icterus. Her abdomen is soft, mildly distended with marked right upper quadrant and epigastric tenderness to palpation, hypoactive bowel sounds, and no masses or organomegaly appreciated. Her stool is negative for occult blood. Laboratory studies are significant for a total bilirubin (9.2 g/dL) with a direct fraction of 4.8 g/dL, alkaline phosphatase 285 IU/L, aspartate aminotransferase (AST) 78 IU/L, alanine aminotransferase (ALT) 92 IU/L, and elevated amylase level 1249 IU/L. Her leukocyte count is 16,500/mm3 with 82% polymorphonuclear cells and 16% lymphocytes. Serum electrolytes, blood urea nitrogen (BUN), and creatinine are normal. A plain film of the abdomen shows a nonspecific gas pattern and no pneumoperitoneum, and chest x-ray is normal. what is the most likely underlying etiology?

choledocholithiasis

A 37-year-old executive returns to your clinic for follow-up of recurrent upper abdominal pain. He initially presented 3 weeks ago, complaining of an increase in frequency and severity of burning epigastric pain, which he has experienced occasionally for more than 2 years. Now the pain occurs three or four times per week, usually when he has an empty stomach, and it often awakens him at night. The pain usually is relieved within minutes by food or over-the-counter antacids, but then recurs within 2 to 3 hours. He admitted that stress at work had recently increased and that because of long working hours, he was drinking more caffeine and eating a lot of take-out foods. His medical history and review of systems were otherwise unremarkable, and, other than the antacids, he takes no medications. His physical examination was normal, including stool guaiac that was negative for occult blood. You advised a change in diet and started him on a proton pump inhibitor. His symptoms resolved completely with the diet changes and daily use of the medication. Results of laboratory tests performed at his first visit show no anemia, but his serum Helicobacter pylori antibody test was positive. What is your next step?

clarithromycin, amoxicillin, PPI, bismuth,

A 48-yo man comes to your office for a routine physical examination. He is a new patient to your practice. He has no significant medical history and takes no medications regularly. His father died at the age of 74 of a heart attack. His mother is alive at the age of 80. She has HTN. He has two younger siblings w/ no known chronic medical conditions. He does not smoke cigarettes, drink alcohol, or use any recreational drugs. He does not exercise. On examination, his BP is 127/82, pulse 80 bpm, respiratory rate 18 breaths/min, ht 67in, wt 90lb. On careful physical exam, no abnormalities are noted. What screening tests for cancer should be performed?

colon ca

•A 40-year-old man presents to the clinic complaining of having 10 episodes of watery, nonbloody diarrhea that started last night. He vomited twice last night but has been able to tolerate liquids today. He has had intermittent abdominal cramps as well. He reports having muscle aches, weakness, headache, and low-grade temperature. He is here with his daughter, who started with the same symptoms this morning. On questioning, he states that he has no significant medical history, no surgeries, and does not take any medications. He does not smoke cigarettes, drink alcohol, use any illicit drugs, and has never had a blood transfusion. He and his family returned to the United States yesterday, following a week-long vacation in Mexico. On examination, he is not in acute distress. His blood pressure is 110/60 mm Hg, his pulse is 98 beats/min, his respiratory rate is 16 breaths/min, and his temperature is 99.1°F (37.2°C). His mucous membranes are dry. His bowel sounds are hyperactive and his abdomen is mildly tender throughout, but there is no rebound tenderness and no guarding. A rectal examination is normal and his stool is guaiac negative. The remainder of his examination is unremarkable. what is the next step?

culture the poop, CBC, IVF (PO fluids if tolerated)

ØAn 83-year-old woman is brought to the clinic by her husband who was concerned with his wife's memory problems. He first noticed some memory decline a few years ago, but the onset was subtle and did not interfere with her day-to-day activities. Mainly, she has some difficulty remembering details, is repeating things, and is being forgetful. ØThe patient's family noticed her gradually increasing memory problems, particularly over the past year. She is unable to remember her appointments and relies heavily on written notes and appointment books. Recently, she got lost while driving and was found by her family 10 hours later. She was unable to use her cell phone and was unsure about her home address and phone number. The patient says that she has always been forgetful. ØHer medical history is significant for well-controlled hypertension and a history of mastectomy secondary to breast cancer diagnosed 20 years ago. She has no significant history of tobacco or alcohol use. ØShe is independent with all activities of daily living, but needs assistance with medication administration, banking, and transportation. She is up-to-date with her health maintenance and immunization. vitals normal. most likely dx?

dementia w/ altzheimers

•A 63-year-old otherwise healthy man presents with a 3-month history of intermittent "dizziness." He describes the episodes as a sensation of "room spinning" and notes that they are brought on or exacerbated by change in position, particularly when rolling out of bed in the morning. Episodes are generally self-limiting, lasting from 10 to 15 seconds on average. He denies associated nausea, vomiting, blurry or double-vision, hearing loss, tinnitus, ear pain, or headache. He does not take any medications. On examination, he is a well-nourished, healthy-appearing man. Vital signs are within normal limits, and he is afebrile. There are no lesions or masses on his face or head. His voice is normal, and speech is fluent. His ear canals and tympanic membranes are normal-appearing. His symptoms can be provoked by lying down and changing position of his head. While symptomatic, he also develops rotational nystagmus. The nystagmus is self-limiting, with the fast phase occurring counterclockwise (from patient's perspective) as his head is tilted 45-degrees to the left (geotropic nystagmus). He improves a few seconds after sitting up, and nystagmus is no longer noted. The remainder of his general and neurologic examination is unremarkable. what is your next diagnositic step?

dix-hallpike maneuver and consider imaging

ØA 72-year-old man is admitted to the hospital because of the acute onset of a right facial droop, right arm weakness, and some difficulty speaking. These symptoms started 6 hours ago while he was sitting at the breakfast table. He had no headache, no diminishment of consciousness, and no abnormal involuntary movements. Two weeks ago, he had a transient painless loss of vision in his left eye, which resolved spontaneously within a few hours. His medical history is significant for long-standing hypertension and a myocardial infarction (MI) 4 years previously, which was treated with percutaneous angioplasty. His medications include a daily aspirin, metoprolol, and simvastatin. He does not smoke. When you see him in the emergency room, his symptoms have nearly resolved. ØHe is afebrile, heart rate is 62 bpm, and blood pressure is 135/87 mm Hg. The corner of his mouth droops, with slight flattening of the right nasolabial fold, but he is able to fully elevate his eyebrows. His strength is 4/5 in his right arm and hand, and the rest of his neurologic examination is normal. He has no carotid bruits, his heart rhythm is regular with no murmur but with an S4 gallop. The remainder of his physical examination is normal. Laboratory studies, including renal function, liver function, lipid profile, and complete blood count (CBC), all are normal. Within a few hours, all of the patient's symptoms have resolved. Name 3 common modifiable risk factors for this diagnosis

don't smoke, don't drink, diet and exercise, modify cholesterol and BP

A 37-year-old executive returns to your clinic for follow-up of recurrent upper abdominal pain. He initially presented 3 weeks ago, complaining of an increase in frequency and severity of burning epigastric pain, which he has experienced occasionally for more than 2 years. Now the pain occurs three or four times per week, usually when he has an empty stomach, and it often awakens him at night. The pain usually is relieved within minutes by food or over-the-counter antacids, but then recurs within 2 to 3 hours. He admitted that stress at work had recently increased and that because of long working hours, he was drinking more caffeine and eating a lot of take-out foods. His medical history and review of systems were otherwise unremarkable, and, other than the antacids, he takes no medications. His physical examination was normal, including stool guaiac that was negative for occult blood. You advised a change in diet and started him on a proton pump inhibitor. His symptoms resolved completely with the diet changes and daily use of the medication. Results of laboratory tests performed at his first visit show no anemia, but his serum Helicobacter pylori antibody test was positive. what is the most likely dx?

duodenal ulcer 2/2 H. pylori

A 46-year-old woman presents to your office complaining of a right hand tremor that has been steadily worsening over the past 2 years. She works as a literary agent and states that this tremor is increasingly impairing her ability to work, since this is affecting her dominant hand. She tells you in a slightly quivering voice, "I am often required to take my clients out to lunch, and I get embarrassed when I cannot eat and drink normally. Sometimes, I cannot even drink from a cup without shaking." She finds that a glass of wine with her meal sometimes helps minimize the tremor. On examination, her blood pressure is 125/85 mm Hg, her pulse is 84 beats/min, and her respiratory rate is 16 breaths/min. Neurologic examination reveals a mild head tremor, but no resting tremor of the hands. When she holds a pen by its tip at arm's length however, a coarse bilateral tremor becomes readily visible, which is more pronounced on the right side. The rest of her examination is unremarkable. what is the most likely dx?

essential tremor

A 48-yo man comes to your office for a routine physical examination. He is a new patient to your practice. He has no significant medical history and takes no medications regularly. His father died at the age of 74 of a heart attack. His mother is alive at the age of 80. She has HTN. He has two younger siblings w/ no known chronic medical conditions. He does not smoke cigarettes, drink alcohol, or use any recreational drugs. He does not exercise. On examination, his BP is 127/82, pulse 80 bpm, respiratory rate 18 breaths/min, ht 67in, wt 90lb. On careful physical exam, no abnormalities are noted. What lifestyle recommendations should be recommended?

exercise (150min/wk with mod exercise), smoke and alcohol (don't do it!), safe sex, seatbelts

ØA 47-year-old African-American man presents to your office for a follow-up visit. He was seen 3 weeks ago for an upper respiratory infection and noted incidentally to have a blood pressure of 164/98 mm Hg. He vaguely remembered being told in the past that his blood pressure was "borderline." He feels fine, has no complaints, and his review of systems is entirely negative. He does not smoke cigarettes, drinks "a couple of beers on the weekends," and does not exercise regularly. He has a sedentary job. His father died of a stroke at the age of 69. His mother is alive and in good health at the age of 72. He has two siblings and is not aware of any chronic medical issues that they have. In the office today, his blood pressure is 156/96 mm Hg in his left arm and 152/98 mm Hg in the right arm. He is afebrile, his pulse is 78 beats/min, respiratory rate 14 breaths/min, he is 70-in tall, and weighs 260 lbs. A general physical examination is normal. what nonpharmacological interventions are beneficial?

exercise, diet, no smoking or drinking

ØA previously healthy 48-year-old accountant presents to his primary care office with severe low back pain that began the previous day after he helped his daughter move into her college dorm. He denies any trauma or previous back injury. He describes the pain as generally "achy," and sometimes characterized as being "sharp" when he moves suddenly. The pain is located in his lower back and radiates down the back of both legs to the middle of his posterior thighs. He has been continent of both bowel and bladder and denies any weakness in his legs. He denies fever, chills, weight loss, or malaise. He finds it very difficult to stand for prolonged periods of time and it has not been relieved with acetaminophen or ibuprofen. His past medical history is significant for hypertension; his only medications are metoprolol and a baby aspirin daily. He does not smoke or use illicit drugs, and only drinks alcohol on occasion. On physical examination, he is well-developed, overweight, and in moderate discomfort. On neuromuscular examination, he has moderate tenderness bilaterally in his lumbar paraspinous muscles, and his lumbar flexion and extension are limited by pain. Strength and sensation are within normal limits and equal and symmetrical bilaterally, as are his deep tendon knee and ankle reflexes. Straight leg raise testing is negative bilaterally and gait is within normal limits. what is the best treatment?

flexeril, dexamethasone, NSAIDs (don't rest but don't make it hurt)

ØA 65-year-old man with benign prostatic hypertrophy had been experiencing difficulty with urination, and so he saw his urologist to be evaluated for a transurethral resection of the prostate. As part of the routine preoperative evaluation, he had a complete blood count, but that was found to be abnormal. The procedure was cancelled and he is now referred to the internal medicine clinic for additional evaluation. ØAside from his prostate symptoms, the patient is asymptomatic. He has not experienced any recent fevers, chills, night sweats, arthralgias, or myalgias. His appetite is good and his weight has been stable. He is moderately physically active, plays golf regularly, and has not noted any fatigue or exertional dyspnea. ØOn examination, he is afebrile and normotensive. His conjunctivae are anicteric, and his skin and oral mucosa show no pallor. His chest is clear to auscultation, and his heart is regular without any murmurs. Abdominal exam is normal, no lymphadenopathy. Laboratories show the following results: white blood cell (WBC) count is 56,000 with 90% mature lymphocytes and 10% neutrophils, hemoglobin is 14.8 g/dL, hematocrit is 45%, and platelet count is 189,000/uL. His CMP is normal. what is the most appropriate step?

flow cytometry

A 48-yo man comes to your office for a routine physical examination. He is a new patient to your practice. He has no significant medical history and takes no medications regularly. His father died at the age of 74 of a heart attack. His mother is alive at the age of 80. She has HTN. He has two younger siblings w/ no known chronic medical conditions. He does not smoke cigarettes, drink alcohol, or use any recreational drugs. He does not exercise. On examination, his BP is 127/82, pulse 80 bpm, respiratory rate 18 breaths/min, ht 67in, wt 90lb. On careful physical exam, no abnormalities are noted. What immunizations should be recommended?

flu and Tdap

ØA 56-year-old white man comes in for a routine health maintenance visit. He is new to your practice and has no specific complaints today. He has hypertension for which he takes hydrochlorothiazide. He has no other significant medical history. He does not smoke cigarettes, occasionally drinks alcohol, and does not exercise. His father died of a heart attack at age 60 and his mother died of cancer at age 72. He has two younger sisters who are in good health. On examination, his blood pressure is 130/80 mm Hg and his pulse is 75 beats/min. He is 6-ft tall and weighs 200 lb. His complete physical examination is normal. You order a fasting lipid panel, which subsequently returns with the following results: total cholesterol 242 mg/dL; triglycerides 138 mg/dL; high-density lipoprotein (HDL) cholesterol 48 mg/dL; and low-density lipoprotein (LDL) cholesterol 155 mg/dL. ØHis 10-year risk of atherosclerotic cardiovascular disease is 9.3% (based on his age, race, nonsmoking status, controlled hypertension, total cholesterol, and HDL level) what other studies should be done?

glc, Cr, LFT, TSH

ØA 47-year-old African-American man presents to your office for a follow-up visit. He was seen 3 weeks ago for an upper respiratory infection and noted incidentally to have a blood pressure of 164/98 mm Hg. He vaguely remembered being told in the past that his blood pressure was "borderline." He feels fine, has no complaints, and his review of systems is entirely negative. He does not smoke cigarettes, drinks "a couple of beers on the weekends," and does not exercise regularly. He has a sedentary job. His father died of a stroke at the age of 69. His mother is alive and in good health at the age of 72. He has two siblings and is not aware of any chronic medical issues that they have. In the office today, his blood pressure is 156/96 mm Hg in his left arm and 152/98 mm Hg in the right arm. He is afebrile, his pulse is 78 beats/min, respiratory rate 14 breaths/min, he is 70-in tall, and weighs 260 lbs. A general physical examination is normal. what is the next step in dx?

glc, K, fasting cholesterol panel, GFR, Cr, Ca, CBC, A/A, ECG, A1c

ØAn 83-year-old woman is brought to the clinic by her husband who was concerned with his wife's memory problems. He first noticed some memory decline a few years ago, but the onset was subtle and did not interfere with her day-to-day activities. Mainly, she has some difficulty remembering details, is repeating things, and is being forgetful. ØThe patient's family noticed her gradually increasing memory problems, particularly over the past year. She is unable to remember her appointments and relies heavily on written notes and appointment books. Recently, she got lost while driving and was found by her family 10 hours later. She was unable to use her cell phone and was unsure about her home address and phone number. The patient says that she has always been forgetful. ØHer medical history is significant for well-controlled hypertension and a history of mastectomy secondary to breast cancer diagnosed 20 years ago. She has no significant history of tobacco or alcohol use. ØShe is independent with all activities of daily living, but needs assistance with medication administration, banking, and transportation. She is up-to-date with her health maintenance and immunization. what lab testing and imaging?

glc, U/A, CT, CBC, LFT, CMP, B12, TSH, CT, MRI,

A 75yo white man presents for a health maintenance checkup. The patient has stable HTN but has not seen a physician in more than 2 years. He denies any particular problem. He lives alone. He takes an aspirin a day and is compliant with his blood pressure medication (hydrochlorothiazide). His son fears that his father is either experiencing a stroke or getting Alzheimer disease because he is having trouble understanding what family members are saying, especially during social events. The son reported no noticeable weakness or gait impairment. On physical examination, the patients, BP was 130/80. Examination of the ears showed no cerumen impaction and normal tympanic membranes. His general examination is normal including his MMSE. Laboratory studies, including TSH are normal. What is the next step?

hearing aids and hearing tests w/ audiologists

ØYou were busy seeing patients in your outpatient clinic when you heard a commotion coming from the waiting room. You went to check and found a very frantic mother and her 2-year-old son who is clutching his throat, coughing, drooling, and visibly struggling to breathe. The mother endorses that just a few minutes ago, the child was running around while eating grapes when she suddenly heard him gagging and wheezing. Her son has an appointment for well-child examination and he is apparently doing well. He has no significant history of respiratory illness. The toddler is still conscious but unable to talk, and his cough is becoming weaker. Breath sounds are decreased bilaterally, with wheezing and stridor heard on auscultation. You tried to ventilate the patient with the chin-lift maneuver but the chest fails to rise. You opened the mouth but you are unable to see any foreign object. what is the next step in management of this patient?

heimlich maneuver

•A 28-year-old man comes to your clinic complaining of a 5-day history of nausea, vomiting, diffuse abdominal pain, fever to 101°F, and muscle aches. He has lost his appetite, but he is able to tolerate liquids and has no diarrhea. He has no significant medical history or family history, and he has not traveled outside the United States. He admits to having 12 different lifetime sexual partners, denies illicit drug use, and drinks alcohol occasionally, but not since this illness began. He takes no medications routinely, but he has been taking acetaminophen, approximately 30 tablets per day for 2 days for fever and body aches since this illness began. On examination, his temperature is 100.8°F, heart rate is 98 bpm, and blood pressure is 120/74 mm Hg. He appears jaundiced, his chest is clear to auscultation, and his heart rhythm is regular without murmurs. His liver percusses 12 cm, and is smooth and slightly tender to palpation. He has no abdominal distention or peripheral edema. Laboratory values are significant for a normal complete blood count, creatinine 1.1 mg/dL, alanine aminotransferase (ALT) 3440 IU/L, aspartate aminotransferase (AST) 2705 IU/L, total bilirubin 24.5 mg/dL, direct bilirubin 18.2 mg/dL, alkaline phosphatase 349 IU/L, serum albumin 3.0 g/dL, and prothrombin time 14 seconds. what is the most likely dx?

hepatitis w/ acetaminophen OD acute liver failure/hepatotoxicity

ØA 58-year-old woman presents to your office for follow-up of an emergency department visit. She was seen 1 week earlier in the emergency department for abdominal pain and was diagnosed with nephrolithiasis. Ultimately, she was sent home with pain medications and given instructions to strain her urine for stones and to follow up with her primary care physician. Today, she is asymptomatic. She takes no medications on a regular basis. Her family history is significant only for a father with high blood pressure. She had several routine laboratory tests drawn in the emergency department, copies of which she brings with her. Upon your review of the laboratory values, you note the following: sodium 142 mEq/L (135-145); potassium 4.0 mEq/L (3.5-5.0); chloride 104 mg/dL (95-105); bicarbonate 28 mEq/L (20-29); blood urea nitrogen (BUN) 20 mg/dL (7-20); creatinine 0.9 mg/dL (0.8-1.4); calcium 12.5 mg/dL (8.5-10.2); albumin 4.2 g/dL (3.4-5.4). The complete blood count (CBC) was within normal limits. ØThe renal calculus was detected by helical computed tomography (CT) scanning without contrast and was located in the right mid ureter. ØYour patient has brought with her the stone that she has strained from the urine. Upon questioning, you learn that she has had multiple episodes of "kidney stones" in the past 2 years. You send the stone to the laboratory for analysis and order a repeat serum calcium level. The results show that the stone is made of calcium oxalate; the serum calcium is still elevated at 11.9 mg/dL. what is the most likely cause?

hyper parathyroidism

ØA 58-year-old woman presents to your office for follow-up of an emergency department visit. She was seen 1 week earlier in the emergency department for abdominal pain and was diagnosed with nephrolithiasis. Ultimately, she was sent home with pain medications and given instructions to strain her urine for stones and to follow up with her primary care physician. Today, she is asymptomatic. She takes no medications on a regular basis. Her family history is significant only for a father with high blood pressure. She had several routine laboratory tests drawn in the emergency department, copies of which she brings with her. Upon your review of the laboratory values, you note the following: sodium 142 mEq/L (135-145); potassium 4.0 mEq/L (3.5-5.0); chloride 104 mg/dL (95-105); bicarbonate 28 mEq/L (20-29); blood urea nitrogen (BUN) 20 mg/dL (7-20); creatinine 0.9 mg/dL (0.8-1.4); calcium 12.5 mg/dL (8.5-10.2); albumin 4.2 g/dL (3.4-5.4). The complete blood count (CBC) was within normal limits. ØThe renal calculus was detected by helical computed tomography (CT) scanning without contrast and was located in the right mid ureter. ØYour patient has brought with her the stone that she has strained from the urine. Upon questioning, you learn that she has had multiple episodes of "kidney stones" in the past 2 years. You send the stone to the laboratory for analysis and order a repeat serum calcium level. The results show that the stone is made of calcium oxalate; the serum calcium is still elevated at 11.9 mg/dL. What is your diagnosis?

hypercalcemia w/ nephrolithiasis

ØA 63-year-old African-American woman is brought to the emergency center for upper arm pain and swelling following a fall at home. The family has noted that for approximately the past 2 months, the patient has become progressively fatigued and absentminded, and she has developed loss of appetite and weight loss. She has been getting up to urinate several times per night and complains of thirst; however, a glucose test for diabetes in her doctor's office was negative. This morning, she lost her balance because she felt "light-headed" and fell, landing on her left arm. Physical examination is notable for an elderly, thin woman in mild distress as a result of pain. She is afebrile, and her blood pressure is 110/70 mm Hg and heart rate is 80 bpm. Her thyroid gland is normal to palpation. Her mucous membranes are somewhat dry and sticky. Heart and lung examinations are normal, and carotid auscultation reveals no bruits. Examination of her extremities is significant only for deformity of the left mid-humerus with swelling. The left radial pulse is 2+ and symmetric. The radiologist calls you to confirm the fracture of the mid-left humerus but also states that there is the suggestion of some lytic lesions of the proximal humerus and recommends a skull film. Serum creatinine level is 2.1 mg/dL, with normal electrolyte and glucose concentrations, but serum calcium level is 13 mg/dL and hemoglobin level is 9.2 g/dL. What is the most likely diagnosis?

hypercalcemia w/ pathologic fx of the left humerus

A 37-year-old previously healthy woman presents to your clinic for unintentional weight loss. Over the past 3 months, she has lost approximately 15 lb without changing her diet or activity level. Otherwise, she feels great. She has an excellent appetite, no gastrointestinal complaints except for occasional loose stools, a good energy level, and no complaints of fatigue. She denies heat or cold intolerance. On examination, her heart rate is 108 bpm, blood pressure 142/82 mm Hg, and she is afebrile. When she looks at you, she seems to stare, and her eyes are somewhat protuberant. You note a large, smooth, nontender thyroid gland and a 2/6 systolic ejection murmur on cardiac examination, and her skin is warm and dry. There is a fine resting tremor. what is the most likely diagnosis?

hyperthyroidism

ØA 50-year-old male with a past medical history of hypertension and a history of palpitations presents for follow-up of his hypertension. He is adherent with a low sodium diet, exercises 150 minutes weekly, and is taking his metoprolol at maximum dose (for the HTN and palpitations). His home blood pressure logs and the clinic reading reveal blood pressures in the range 140-150/90-100. The rest of his vitals including heart rate are normal, as his examination. You decide to add a thiazide diuretic to his existing antihypertensive regimen. what electrolyte imbalances with thiazides?

hyponatremia, hypokalemia, hypochloridemia, hypercalcemia

ØA 50-year-old male with a past medical history of hypertension and a history of palpitations presents for follow-up of his hypertension. He is adherent with a low sodium diet, exercises 150 minutes weekly, and is taking his metoprolol at maximum dose (for the HTN and palpitations). His home blood pressure logs and the clinic reading reveal blood pressures in the range 140-150/90-100. The rest of his vitals including heart rate are normal, as his examination. You decide to add a thiazide diuretic to his existing antihypertensive regimen. what is the mechanism of action of thiazide?

inhibit NaCl in DCT by inhibit NaCl transporter

ØA 48-year-old man comes to your office with a 6-day history of worsening cough productive of green sputum. He has had fever and chills. He complains of pain in the right midback with deep breathing or coughing. Further history reveals that he has smoked one pack of cigarettes a day for 30 years. He has no other significant medical history. On examination, his temperature is 38.1°C (100.5°F); his respiratory rate is 24 breaths per minute; pulse, 98 beats per minute; blood pressure, 120/75 mm Hg; and saturation of oxygen, 96 percent on room air by pulse oximetry. Auscultation of his lungs reveals rales in the right lower-posterior lung field. The remainder of his examination is within normal limits. A posterior-to-anterior (PA) and lateral chest x-ray show a right lower-lobe infiltrate. A sputum Gram stain reveals gram-positive cocci, and subsequent sputum and blood culture results confirm the diagnosis of pneumonia caused by Streptococcus pneumoniae (pneumococcus). You treat him with a combination of amoxicillin and clavulanic acid. what is the mechanism of action of amoxicillin?

inhibits bacterial cell wall to rupture

•A 63-year-old otherwise healthy man presents with a 3-month history of intermittent "dizziness." He describes the episodes as a sensation of "room spinning" and notes that they are brought on or exacerbated by change in position, particularly when rolling out of bed in the morning. Episodes are generally self-limiting, lasting from 10 to 15 seconds on average. He denies associated nausea, vomiting, blurry or double-vision, hearing loss, tinnitus, ear pain, or headache. He does not take any medications. On examination, he is a well-nourished, healthy-appearing man. Vital signs are within normal limits, and he is afebrile. There are no lesions or masses on his face or head. His voice is normal, and speech is fluent. His ear canals and tympanic membranes are normal-appearing. His symptoms can be provoked by lying down and changing position of his head. While symptomatic, he also develops rotational nystagmus. The nystagmus is self-limiting, with the fast phase occurring counterclockwise (from patient's perspective) as his head is tilted 45-degrees to the left (geotropic nystagmus). He improves a few seconds after sitting up, and nystagmus is no longer noted. The remainder of his general and neurologic examination is unremarkable. what are other treatments for this condition?

meclizine, diazepam, transdermal scopolamine

ØA 63-year-old African-American woman is brought to the emergency center for upper arm pain and swelling following a fall at home. The family has noted that for approximately the past 2 months, the patient has become progressively fatigued and absentminded, and she has developed loss of appetite and weight loss. She has been getting up to urinate several times per night and complains of thirst; however, a glucose test for diabetes in her doctor's office was negative. This morning, she lost her balance because she felt "light-headed" and fell, landing on her left arm. Physical examination is notable for an elderly, thin woman in mild distress as a result of pain. She is afebrile, and her blood pressure is 110/70 mm Hg and heart rate is 80 bpm. Her thyroid gland is normal to palpation. Her mucous membranes are somewhat dry and sticky. Heart and lung examinations are normal, and carotid auscultation reveals no bruits. Examination of her extremities is significant only for deformity of the left mid-humerus with swelling. The left radial pulse is 2+ and symmetric. The radiologist calls you to confirm the fracture of the mid-left humerus but also states that there is the suggestion of some lytic lesions of the proximal humerus and recommends a skull film. Serum creatinine level is 2.1 mg/dL, with normal electrolyte and glucose concentrations, but serum calcium level is 13 mg/dL and hemoglobin level is 9.2 g/dL. what is the most likely underlying etiology in this patient?

multiple myeloma (myeloid cells)

ØA 72-year-old man is admitted to the hospital because of the acute onset of a right facial droop, right arm weakness, and some difficulty speaking. These symptoms started 6 hours ago while he was sitting at the breakfast table. He had no headache, no diminishment of consciousness, and no abnormal involuntary movements. Two weeks ago, he had a transient painless loss of vision in his left eye, which resolved spontaneously within a few hours. His medical history is significant for long-standing hypertension and a myocardial infarction (MI) 4 years previously, which was treated with percutaneous angioplasty. His medications include a daily aspirin, metoprolol, and simvastatin. He does not smoke. When you see him in the emergency room, his symptoms have nearly resolved. ØHe is afebrile, heart rate is 62 bpm, and blood pressure is 135/87 mm Hg. The corner of his mouth droops, with slight flattening of the right nasolabial fold, but he is able to fully elevate his eyebrows. His strength is 4/5 in his right arm and hand, and the rest of his neurologic examination is normal. He has no carotid bruits, his heart rhythm is regular with no murmur but with an S4 gallop. The remainder of his physical examination is normal. Laboratory studies, including renal function, liver function, lipid profile, and complete blood count (CBC), all are normal. Within a few hours, all of the patient's symptoms have resolved. ØWhat is the next step?

non-contrast CT

ØA previously healthy 48-year-old accountant presents to his primary care office with severe low back pain that began the previous day after he helped his daughter move into her college dorm. He denies any trauma or previous back injury. He describes the pain as generally "achy," and sometimes characterized as being "sharp" when he moves suddenly. The pain is located in his lower back and radiates down the back of both legs to the middle of his posterior thighs. He has been continent of both bowel and bladder and denies any weakness in his legs. He denies fever, chills, weight loss, or malaise. He finds it very difficult to stand for prolonged periods of time and it has not been relieved with acetaminophen or ibuprofen. His past medical history is significant for hypertension; his only medications are metoprolol and a baby aspirin daily. He does not smoke or use illicit drugs, and only drinks alcohol on occasion. On physical examination, he is well-developed, overweight, and in moderate discomfort. On neuromuscular examination, he has moderate tenderness bilaterally in his lumbar paraspinous muscles, and his lumbar flexion and extension are limited by pain. Strength and sensation are within normal limits and equal and symmetrical bilaterally, as are his deep tendon knee and ankle reflexes. Straight leg raise testing is negative bilaterally and gait is within normal limits. what is the workup?

none really needed until 1mo

•A 4-year-old child is seen by the pediatrician for ear pain. His mother reports he was in his usual state of good health until about 5 days ago when he developed an upper respiratory infection (URI) consisting of clear nasal discharge and cough. He had been having normal activity and intake until about 48 hours prior when he developed a temperature of 102.1°F (38.9°C) and complaints that his left ear hurts. She denies nausea, vomiting, diarrhea, headache, or change in urine output. She reports that he did not sleep well the previous evening, awakening several times complaining of ear pain, but remains somewhat interested in his toys earlier in the day. What is the diagnosis?

otitis media

ØA 26-year-old woman presents to the emergency center on a Saturday afternoon complaining of bleeding from her nose and mouth since the previous night. She also noticed small, reddish spots on her lower extremities when she got out of the bed this morning. She denies fever, chills, nausea, vomiting, abdominal pain, or joint pain. The patient reports she had developed an upper respiratory infection 2 weeks prior to the emergency room visit, but the infection has now resolved. She denies significant medical problems. Her menses have been normal, and her last menstrual period was approximately 2 weeks ago. She denies excessive bleeding in the past, even after delivering her baby. Prior to this episode, she never had epistaxis, easy bruisability, or bleeding into her joints. There is no family history of abnormal bleeding. The patient does not take any medications. On examination she is alert, oriented, and somewhat anxious. Her blood pressure is 110/70 mm Hg, her heart rate is 90 bpm, and she is afebrile. No pallor or jaundice is noted. There is bright red oozing from the nose and the gingiva. Skin examination reveals multiple 1-mm flat reddish spots on her lower extremities. The rest of the examination is normal. There is no lymphadenopathy or hepatosplenomegaly. Her complete blood cell count (CBC) is normal except for a platelet count of 18,000/mm3. Prothrombin time (PT) and partial thromboplastin time (PTT) are normal. what is the next diagnostic step?

peripheral smear

A 75yo white man presents for a health maintenance checkup. The patient has stable HTN but has not seen a physician in more than 2 years. He denies any particular problem. He lives alone. He takes an aspirin a day and is compliant with his blood pressure medication (hydrochlorothiazide). His son fears that his father is either experiencing a stroke or getting Alzheimer disease because he is having trouble understanding what family members are saying, especially during social events. The son reported no noticeable weakness or gait impairment. On physical examination, the patients, BP was 130/80. Examination of the ears showed no cerumen impaction and normal tympanic membranes. His general examination is normal including his MMSE. Laboratory studies, including TSH are normal. What is the most likely diagnosis?

presbycusis

ØA 48-year-old man comes to your office with a 6-day history of worsening cough productive of green sputum. He has had fever and chills. He complains of pain in the right midback with deep breathing or coughing. Further history reveals that he has smoked one pack of cigarettes a day for 30 years. He has no other significant medical history. On examination, his temperature is 38.1°C (100.5°F); his respiratory rate is 24 breaths per minute; pulse, 98 beats per minute; blood pressure, 120/75 mm Hg; and saturation of oxygen, 96 percent on room air by pulse oximetry. Auscultation of his lungs reveals rales in the right lower-posterior lung field. The remainder of his examination is within normal limits. A posterior-to-anterior (PA) and lateral chest x-ray show a right lower-lobe infiltrate. A sputum Gram stain reveals gram-positive cocci, and subsequent sputum and blood culture results confirm the diagnosis of pneumonia caused by Streptococcus pneumoniae (pneumococcus). You treat him with a combination of amoxicillin and clavulanic acid. what is the mechanism of action of clavulanic acid?

prevents betalactam breakdown

A 46-year-old woman presents to your office complaining of a right hand tremor that has been steadily worsening over the past 2 years. She works as a literary agent and states that this tremor is increasingly impairing her ability to work, since this is affecting her dominant hand. She tells you in a slightly quivering voice, "I am often required to take my clients out to lunch, and I get embarrassed when I cannot eat and drink normally. Sometimes, I cannot even drink from a cup without shaking." She finds that a glass of wine with her meal sometimes helps minimize the tremor. On examination, her blood pressure is 125/85 mm Hg, her pulse is 84 beats/min, and her respiratory rate is 16 breaths/min. Neurologic examination reveals a mild head tremor, but no resting tremor of the hands. When she holds a pen by its tip at arm's length however, a coarse bilateral tremor becomes readily visible, which is more pronounced on the right side. The rest of her examination is unremarkable. pharmacotherapy?

propranolol and primidone

A 37-year-old previously healthy woman presents to your clinic for unintentional weight loss. Over the past 3 months, she has lost approximately 15 lb without changing her diet or activity level. Otherwise, she feels great. She has an excellent appetite, no gastrointestinal complaints except for occasional loose stools, a good energy level, and no complaints of fatigue. She denies heat or cold intolerance. On examination, her heart rate is 108 bpm, blood pressure 142/82 mm Hg, and she is afebrile. When she looks at you, she seems to stare, and her eyes are somewhat protuberant. You note a large, smooth, nontender thyroid gland and a 2/6 systolic ejection murmur on cardiac examination, and her skin is warm and dry. There is a fine resting tremor. what are the options for treatment?

radioactive iodine ablation, surgery, propranolol for tachy, PTU/methimazole

ØA 25-year-old African-American man is admitted to your service with the diagnosis of a sickle cell pain episode. He was admitted to the hospital six times last year with the same diagnosis, and he was last discharged 2 months ago. This time, he presented to the emergency department complaining of abdominal and bilateral lower extremity pain, his usual sites of pain. When you examine him, you note he is febrile to 101°F, with respiratory rate 25 bpm, normal blood pressure, HR 100 bpm. Lung examination reveals bronchial breath sounds and egophony in the right lung base. His oxygen saturation on 2 L/min nasal cannula is 92%. Besides the usual abdominal and leg pain, he is now complaining of chest pain, which is worse on inspiration. Although he is tender on palpation of his extremities, the remainder of his examination is normal. His laboratory examinations reveal elevated white blood cell and reticulocyte counts, and a hemoglobin and hematocrit that are slightly lower than baseline. The peripheral smear is shown. complications?

resp and possible death

A 35-year-old woman with a history of asthma presents to your office with symptoms of nasal itching, sneezing, and rhinorrhea. She states she feels this way most days, but her symptoms are worse in the spring and fall. She has had difficulty sleeping because she is always congested. She states she has taken diphenhydramine (Benadryl) with no relief. She does not smoke cigarettes and does not have exposure to passive smoke, but she does have two cats at home. On examination, she appears tired but is in no respiratory distress. Her vital signs are as follows: temperature 98.8 °F, blood pressure 128/84 mm Hg, pulse 88 beats/min, and respiratory rate 18 breaths/min. The mucosa of her nasal turbinates appears swollen (boggy) and has a pale, bluish-gray color. Thin and watery secretions are seen. No abnormalities are seen on ear examination. There is no cervical lymphadenopathy noted, and her lungs are clear. what are important considerations and potential complications?

season, cat allergy, rebound congestion, pollution, comorbidity complications

ØA 61-year-old woman presents to the emergency room (ER) complaining of cough for 2 weeks. The cough is productive of green sputum and is associated with sweating, shaking chills, and fever up to 102°F (38.8°C). She was exposed to her grandchildren who were told that they had upper respiratory infections 2 weeks ago but now are fine. Her past medical history is significant for diabetes for 10 years, which is under good control using oral hypoglycemics. She denies tobacco, alcohol, or drug use. On examination, she looks ill and in distress, with continuous coughing and chills. Her blood pressure is 100/80 mm Hg, her pulse is 110 beats/min, her temperature is 101°F (38.3°C), her respirations are 24 breaths/min, and her oxygen saturation is 97% on room air. Examination of the head and neck is unremarkable. Her lungs have rales and decreased breath sounds, with dullness to percussion in bilateral bases. Her heart is tachycardic but regular. Her extremities are without signs of cyanosis or edema. The remainder of her examination is normal. A complete blood count (CBC) shows a high white blood cell (WBC) count of 17,000 cells/mm3, with a differential of 85% neutrophils and 20% lymphocytes. Her blood sugar is 120 mg/dL. what are the potential complications for this condition?

sepsis, pleural effusion, empyema, bacteremia

•A 28-year-old man comes to your clinic complaining of a 5-day history of nausea, vomiting, diffuse abdominal pain, fever to 101°F, and muscle aches. He has lost his appetite, but he is able to tolerate liquids and has no diarrhea. He has no significant medical history or family history, and he has not traveled outside the United States. He admits to having 12 different lifetime sexual partners, denies illicit drug use, and drinks alcohol occasionally, but not since this illness began. He takes no medications routinely, but he has been taking acetaminophen, approximately 30 tablets per day for 2 days for fever and body aches since this illness began. On examination, his temperature is 100.8°F, heart rate is 98 bpm, and blood pressure is 120/74 mm Hg. He appears jaundiced, his chest is clear to auscultation, and his heart rhythm is regular without murmurs. His liver percusses 12 cm, and is smooth and slightly tender to palpation. He has no abdominal distention or peripheral edema. Laboratory values are significant for a normal complete blood count, creatinine 1.1 mg/dL, alanine aminotransferase (ALT) 3440 IU/L, aspartate aminotransferase (AST) 2705 IU/L, total bilirubin 24.5 mg/dL, direct bilirubin 18.2 mg/dL, alkaline phosphatase 349 IU/L, serum albumin 3.0 g/dL, and prothrombin time 14 seconds. what is the most important immediate dx test?

serum tylenol levels

A 44-year-old man presents with sudden onset of shaking chills, fever, and productive cough. He was in his usual state of good health until 1 week ago, when he developed mild nasal congestion and achiness. He otherwise felt well until last night, when he became fatigued and feverish, and developed a cough associated with right-sided pleuritic chest pain. His medical history is remarkable only for his 15-pack per year smoking habit. In your office, his vital signs are normal except for a temperature of 102°F. His oxygen saturation on room air is 100%. He is comfortable, except when he coughs. His physical examination is unremarkable except for bronchial breath sounds and end-inspiratory crackles in the right lower lung field. There is a right lower lobe consolidation on chest x-ray. what risk factors are common for this condition?

smoking, occupational, resp comorbidity, EtOH, recent influenza infxn

ØA 65-year-old man with benign prostatic hypertrophy had been experiencing difficulty with urination, and so he saw his urologist to be evaluated for a transurethral resection of the prostate. As part of the routine preoperative evaluation, he had a complete blood count, but that was found to be abnormal. The procedure was cancelled and he is now referred to the internal medicine clinic for additional evaluation. ØAside from his prostate symptoms, the patient is asymptomatic. He has not experienced any recent fevers, chills, night sweats, arthralgias, or myalgias. His appetite is good and his weight has been stable. He is moderately physically active, plays golf regularly, and has not noted any fatigue or exertional dyspnea. ØOn examination, he is afebrile and normotensive. His conjunctivae are anicteric, and his skin and oral mucosa show no pallor. His chest is clear to auscultation, and his heart is regular without any murmurs. Abdominal exam is normal, no lymphadenopathy. Laboratories show the following results: white blood cell (WBC) count is 56,000 with 90% mature lymphocytes and 10% neutrophils, hemoglobin is 14.8 g/dL, hematocrit is 45%, and platelet count is 189,000/uL. His CMP is normal. what is seen on peripheral blood smear?

smudge cells

ØA 37-year-old man presents to your office with the complaint of cough. The cough began approximately 3 months prior to this appointment, and it has become more annoying to the patient. The cough is nonproductive and worse at night and after exercise. He has had a sedentary lifestyle but recently started an exercise program, including jogging, and says he is having a much harder time with the exertion. He runs out of breath earlier than he did to previously, and he coughs a lot. He has not had any fever, blood-tinged sputum, or weight loss. He denies nasal congestion and headaches. He does not smoke and has no significant medical history. His examination is notable for a blood pressure of 134/78 mm Hg and lung findings of occasional expiratory wheezes on forced expiration. A chest radiograph is read as normal. what is the test to confirm the diagnosis?

spirometry w/ reversibility using a bronchodilator

ØA 56-year-old white man comes in for a routine health maintenance visit. He is new to your practice and has no specific complaints today. He has hypertension for which he takes hydrochlorothiazide. He has no other significant medical history. He does not smoke cigarettes, occasionally drinks alcohol, and does not exercise. His father died of a heart attack at age 60 and his mother died of cancer at age 72. He has two younger sisters who are in good health. On examination, his blood pressure is 130/80 mm Hg and his pulse is 75 beats/min. He is 6-ft tall and weighs 200 lb. His complete physical examination is normal. You order a fasting lipid panel, which subsequently returns with the following results: total cholesterol 242 mg/dL; triglycerides 138 mg/dL; high-density lipoprotein (HDL) cholesterol 48 mg/dL; and low-density lipoprotein (LDL) cholesterol 155 mg/dL. ØHis 10-year risk of atherosclerotic cardiovascular disease is 9.3% (based on his age, race, nonsmoking status, controlled hypertension, total cholesterol, and HDL level). what initial management of his elevated cholesterol?

statin, diet, exercise (risk for CHD)

A 58 yo man comes to your office, because of shortness of breath. He has experienced mild dyspnea on exertion for a few years, but more recently he has noted worsening SOB w/ minimal exercise and the onset of dyspnea at rest. He has difficulty reclining; as a result, he spends the night sitting up in a chair trying to sleep. He reports a cough w/ production of yellowish-brown sputum every morning throughout the year. he denies chest pain, ever, chills, or lower extremity edema. He smoked about two packs of cigarettes per day since age 15. He does not drink alcohol. A few months ago, the patient went to an urgent care clinic for evaluation of his symptoms, and he received a prescription for some inhalers, the names of which he does not remember. He was also told to find a primary care physician for further evaluation. On physical exam, his BP is 135/85, pulse 96bpm, resp 28 bpm, and temp 97.6. He is sitting in a chair, leaning forward w/ his arms braced on his knees. He appears uncomfortalbe w/ labored resp and cyanotic lips. He is using accessory muscles of respiration and chest examinatino reveals wheezes and rhonchi BL, but no crackles. the AP of the chest wall appears increased. Cardiovascular examination reveals distant heart sounds but with a RRR. JVD is NL. Extremities w/o cyanosis, edema, or clubbing. what is the best initial tx?

steroid, beta agonist, O2, antibiotic, loop diuretic

A 44-year-old man presents with sudden onset of shaking chills, fever, and productive cough. He was in his usual state of good health until 1 week ago, when he developed mild nasal congestion and achiness. He otherwise felt well until last night, when he became fatigued and feverish, and developed a cough associated with right-sided pleuritic chest pain. His medical history is remarkable only for his 15-pack per year smoking habit. In your office, his vital signs are normal except for a temperature of 102°F. His oxygen saturation on room air is 100%. He is comfortable, except when he coughs. His physical examination is unremarkable except for bronchial breath sounds and end-inspiratory crackles in the right lower lung field. There is a right lower lobe consolidation on chest x-ray. what is the most likely etiology of this condition?

strep pneumo

ØAn 18-month-old girl is brought to the office by her mother because of a rash. The patient has had a subjective high fever for the past 3 days, along with some mild respiratory symptoms of cough and rhinorrhea. She was given acetaminophen for the fever but no other medications. The fever has gone down in the past day, but today she developed an erythematous rash that developed suddenly, starting on the trunk and spreading to the extremities. The child has no significant medical history and no known sick contacts, although she attends day care 3 days a week. On examination, she is mildly fussy but is easily consolable in her mother's lap. Upon inspection, the rash consists of small macules and papules that blanch on palpation. The remainder of her examination is unremarkable. what is the treatment?

supportive care, rash resolve 24-48hr

•A 45-year-old white woman presents to your office concerned about a "mole" on her face. She says that it has been present for years but her husband has been urging her to have it checked. She denies any pain, itching, or bleeding from the site. She has no significant past medical history, takes no medications, and has no allergies. She has no history of skin cancer in her family. She is an accountant by occupation. •On examination, the patient is normotensive, afebrile, and appears slightly younger than her stated age. A skin examination reveals a nontender, symmetric, 4-mm papule that is uniformly reddish-brown in color. The lesion is well circumscribed, and the surrounding skin is normal in appearance. There are no other lesions in the area. what features are reassuring of benign condition?

symmetric, defined borders, uniform color, not evolving or elevated

ØA 32-year-old woman presents to the emergency center complaining of productive cough, fever, and chest pain for 4 days. She was seen 2 days ago in her primary care physician's clinic with the same complaints, was diagnosed clinically with pneumonia, and was sent home with oral azithromycin. Since then, her cough has diminished in quantity. However, the fever has not abated, and she still experiences left-sided chest pain, which is worse when she coughs or takes a deep breath. In addition, she has started to feel short of breath when she walks around the house. She has no other medical history. She does not smoke and has no history of occupational exposure. She has not traveled outside of the United States and has no sick contacts. ØOn physical examination, her temperature is 103.4°F, heart rate is 116 bpm, blood pressure is 128/69 mm Hg, respiratory rate is 24 bpm and is shallow. Her pulse oximetry is 94% saturation on room air. Physical examination is significant for decreased breath sounds in the lower half of the left lung fields posteriorly, with dullness to percussion about halfway up. There are a few inspiratory crackles in the mid-lung fields, and her right side is clear to auscultation. Her heart is tachycardic but regular with no murmurs. She has no cyanosis. What is the next diagnostic test?

thoracentesis w/ light's criteria

A 37-year-old previously healthy woman presents to your clinic for unintentional weight loss. Over the past 3 months, she has lost approximately 15 lb without changing her diet or activity level. Otherwise, she feels great. She has an excellent appetite, no gastrointestinal complaints except for occasional loose stools, a good energy level, and no complaints of fatigue. She denies heat or cold intolerance. On examination, her heart rate is 108 bpm, blood pressure 142/82 mm Hg, and she is afebrile. When she looks at you, she seems to stare, and her eyes are somewhat protuberant. You note a large, smooth, nontender thyroid gland and a 2/6 systolic ejection murmur on cardiac examination, and her skin is warm and dry. There is a fine resting tremor. how would you confirm the diagnosis?

thyroid panel (TSH, T3, T4), radioactive iodine uptake

A 55yo caucasian woman, new to your practice, presents for an "annual physical examination." She reports that she is healthy, generally feels well, and has no specific complaints. She has a history of having had a "total hysterectomy," by which she means that her uterus, cervix, ovaries, and fallopian tubes were removed. The surgery was performed because of fibroids. She has had a Pap smear every 3 years since the age of 21, all of which have been normal. She has no other significant medical or surgical history. She takes a multivitamin pill daily but no other medications. Her family history is significant for breast cancer that was diagnosed in her maternal grandmother at the age of 72. The patient is married, monogamous, and does not smoke cigarettes or drink alcohol. She tries to avoid dairy products because of "lactose intolerance." She walks 30min a day five times a week for exercise. Her physical examination is normal. What could you recommend to reduce her risk of developing osteoporosis?

vit D and body weight workouts. maybe calcium if no risk of kidney stones

ØA 58-year-old woman presents to your office for follow-up of an emergency department visit. She was seen 1 week earlier in the emergency department for abdominal pain and was diagnosed with nephrolithiasis. Ultimately, she was sent home with pain medications and given instructions to strain her urine for stones and to follow up with her primary care physician. Today, she is asymptomatic. She takes no medications on a regular basis. Her family history is significant only for a father with high blood pressure. She had several routine laboratory tests drawn in the emergency department, copies of which she brings with her. Upon your review of the laboratory values, you note the following: sodium 142 mEq/L (135-145); potassium 4.0 mEq/L (3.5-5.0); chloride 104 mg/dL (95-105); bicarbonate 28 mEq/L (20-29); blood urea nitrogen (BUN) 20 mg/dL (7-20); creatinine 0.9 mg/dL (0.8-1.4); calcium 12.5 mg/dL (8.5-10.2); albumin 4.2 g/dL (3.4-5.4). The complete blood count (CBC) was within normal limits. ØThe renal calculus was detected by helical computed tomography (CT) scanning without contrast and was located in the right mid ureter. ØYour patient has brought with her the stone that she has strained from the urine. Upon questioning, you learn that she has had multiple episodes of "kidney stones" in the past 2 years. You send the stone to the laboratory for analysis and order a repeat serum calcium level. The results show that the stone is made of calcium oxalate; the serum calcium is still elevated at 11.9 mg/dL. what is the next step?

vit D supplementation, calcitonin

A 55yo man comes into your office for f/u of a chronic cough. He also complains of SOB w/ activity. He reports that this has been getting worse over time. As you are interviewing the pt, you note that he smells of cigarette smoke. Upon further question, he reports smoking 1 pack of cigarettes per day for the past 35 years and denies ever being advised to quit. On exam, he is in no resp distress at rest, VS are NL, and no obvious cyanosis. His pulmonary examination is notable for reduced air movement and faint expiratory wheezing on auscultation. What interventions are available to aid w/ smoking cessation?

wellbutrin, 1800QUITNOW


संबंधित स्टडी सेट्स

Chap 12 Basics of Diagnostic Coding

View Set

Health assessment exam 2 - chapter 19

View Set

World History- Chapter 4: Ancient Greece

View Set

1. Basic Insurance Concept and Principles

View Set

My Brother Sam is Dead Character Quotes

View Set

ISSA Personal Trainer Certification - Unit 02 - Basic Anatomy and Physiology

View Set